Você está na página 1de 91

MA 21 Resoluo de Problemas

Professor: Sandro Rodrigues Mazorche




Grupo: 2





Mrcio Eduardo Primo
Braslio Freitas
Paulo Geovane
Paulo Fernandes
Marcelo de Moura Costa















Captulo 1 Solues dos Exerccios
1. Uma sacola contm meias cujas cores so branca, preta, amarela e azul. Sem olhar para a sacola,
qual a quantidade mnima de meias que precisamos retirar da mesma para garantir pelo menos um
par de meias da mesma cor?

Precisamos retirar no mnimo 5 meias. Como o nmero de meias ser maior que o nmero de
cores teremos certeza absoluta que formaremos um par de mesma cor.

2. O pai do padre filho nico de meu pai. O que eu sou do padre?

Como o filho nico do meu pai sou eu, ento, sou o pai do padre.

3. Numa mesa h 5 cartas:
Q T 3 4 6

Cada carta tem de um lado um nmero natural e do outro lado uma letra. Joo afirma: "Qualquer carta
que tenha uma vogal tem um nmero par do outro lado". Pedro provou que Joo mente virando
somente uma das cartas. Qual das 5 cartas foi a que Pedro virou?

Pedro virou a carta 3, mostrando que havia uma vogal escrita no verso.

4. A polcia prende 4 homens, um dos quais culpado de um furto.
Eles fazem as seguintes declaraes:
_ Arnaldo: Bernaldo o culpvel.
_ Bernaldo: Cernaldo o culpvel.
_ Dernaldo: eu no sou culpvel.
_ Cernaldo: Bernaldo mente ao dizer que eu sou culpvel.
S uma destas declaraes a verdadeira, quem culpvel pelo furto?

Testando cada uma das afirmaes, temos que ter todas as outras falsas.
Suponha ento:

(i) Arnaldo fala a verdade. Ento: Bernaldo o culpvel. Nesse caso:
1. Benaldo mente. (Falso)
2. Dernaldo fala a verdade. Isso suficiente para descartar que Arnaldo fale a verdade.

(ii) Bernaldo fala a verdade. Ento: Cernaldo o culpvel. Nesse caso:
1. Arnaldo mente. (Falso)
2. Dernaldo fala a verdade. Descartamos ser Bernaldo que falou a verdade.

(iii) Dernaldo fala a verdade. Ento: ele no culpvel.
1. Arnaldo pode estar falando verdade ou no
2. Bernaldo pode estar falando verdade ou no
3. Cernaldo pode estar falando verdade ou no. Porm, se a afirmao de Cernaldo for verdadeira a
de Bernaldo necessariamente falsa e vice versa. Ou seja, uma das duas declaraes, de Cernaldo
ou Bernaldo tambm ser verdadeira, o que descara a possibilidade de que Dernaldo seja o nico que
fala a verdade.

(iv) Cernaldo fala a verdade. S restou essa afirmao como possivelmente verdadeira. Nesse caso:
Bernaldo mente, e Cernaldo no culpvel.
1. Arnaldo pode estar falando a verdade ou no
2. Bernaldo mente (hiptese de que Cernaldo fale a verdade)
3. Dernaldo pode estar falando a verdade ou no
Conclumos que apenas Cernaldo pode ter falado a verdade, restando que todos outros mentem.
Quem seria o culpvel?
No Bernaldo e nem Cernaldo, pois Arnaldo e Bernaldo falam mentira.
Dernaldo tambm mente, ento necessariamente ele o culpvel.

5. Numa cidade existe uma pessoa X que sempre mente teras, quintas e sbados e completamente
sincera o resto dos dias da semana. Felipe chega um certo dia na cidade e mantm o seguinte dilogo
com a pessoa X:
_ Felipe: Que dia hoje?
_ X: Sbado.
_ Felipe: Que dia ser amanh?
_ X: Quarta-feira.
Em qual dia da semana foi mantido este dilogo?

X necessariamente est no dia de mentir, pois, no possvel hoje ser sbado e amanh ser
quarta-feira, portanto, pode ser tera, quinta ou sbado. Se fosse tera-feira, a afirmao Que dia ser
amanh? seria verdadeira. Se fosse Sbado a afirmao Que dia hoje? seria verdadeira. Portanto,
o dilogo s pode ter acontecido na quinta-feira.

6. Divida o relgio de parede abaixo em 6 partes iguais de forma tal que a soma das horas que ficam
em cada parte seja a mesma.


A soma dos valores: 1 + 2 + ... + 11 + 12 = 78. Dividindo por 6 temos
quociente 13. Portanto devemos ter 6 partes de soma 13. Teremos os pares
(12, 1); (11, 2); (10, 3); (9, 4); (8, 5) e (6, 7).
Para realizar a diviso em partes iguais deve ser procurada uma
disposio geomtrica que satisfaa essa condio.


7. Joo adora Gabriela, que uma aluna excelente em Matemtica. Joo armou um plano para dar um
beijo nela, e descobriu que poder fazer isso apenas dizendo uma frase. Que frase essa?

"Se gostas de Matemtica deves me dar um beijo, mas se no gostas eu a beijarei".


8. No plano se colocam 187 rodas dentadas do mesmo dimetro, enumeradas de 1 at 187. A roda 1
acoplada com a roda 2, a 2 com a 3, : : : , a 186 com a 187 e esta ltima com a roda 1. Pode tal
sistema girar?

As rodas mpares giram em um sentido e as rodas pares no sentido contrrio. Como a roda 187
ser acoplada na roda 1 (ambas mpares) no ser possvel que o sistema gire.


9. Um canal, em forma quadrada, de 4 metros de largura rodeia um castelo. A ponte do castelo est
fechada e um intruso quer entrar no castelo usando duas pranchas de 3,5 metros de comprimento.
Ser que o intruso consegue?


Sendo: AK = AL temos que AM = 1,75 m

Como AB = 2 4 m

MB = 2 4 1,75 ~ 3,91 m

Como a outra prancha tem 3,5 m no possvel atravessar o fosso.
10. Os nmeros 1, 2, 3, . . . ,99 so escritos no quadro-negro e permitido realizar a seguinte
operao: apagar dois deles e substitu-los pela diferena do maior com o menor. Fazemos esta
operao sucessivamente at restar apenas um ltimo nmero no quadro. Pode o ltimo nmero que
restou ser o zero?

Na sequncia 1, 2, ... , 99 exclu-se o 1 e subtra-se os nmeros consecutivos dois a dois,
obteremos 50 nmeros 1 (25 pares) que subtrados dois a dois, resulta zero.

11. Algum elege dois nmeros, no necessariamente distintos, no conjunto de nmeros naturais 2, ...,
20. O valor da soma destes nmeros dado somente a Adriano (A) e o valor do produto dos nmeros
dado unicamente a Karla (K).
_ Pelo telefone A diz a K: _No possvel que descubras minha soma.
_ Uma hora mais tarde, K diz a A: _Ah! Sabendo disso, j sei quanto vale tua soma!
_ Mais tarde A chama outra vez K e lhe informa: _Poxa, agora eu tambm conheo teu produto!
Quais nmeros foram eleitos?

No conseguimos resolver!

Na revista EUREKA! n
o
23 de 2006 foi publicada uma resoluo desse problema pelo Professor
Cssio Neri do Instituto de Matemtica da Universidade Federal do Rio de Janeiro.
O artigo tem por ttulo "O Problema Impossvel" e cita que a forma original foi proposta por
Hans Freudenthal e popularizada por Martin Gardner.
Para solucionar, o Professor Cssio usa um algoritmo computacional em linguagem C
(disponvel em www.labma.ufrj.br/~cassio/f-impossivel.html) e um mtodo no computacional.
Aps as consideraes, em ambos os casos, ele determina que os nmeros eleitos so 4 e 13.


12. possvel cobrir um tabuleiro de xadrez com 31 domins onde removemos as casas dos vrtices
superior esquerdo e inferior direito?

No pois as casas retiradas esto na mesma diagonal, so casas de mesma cor.

13. Num saco encontram-se 64 moedas leves e 64 moedas pesadas. possvel separar duas moedas
de pesos diferentes com 7 pesagens?

Inicialmente dividimos as moedas em dois grupos de 64 moedas. Colocamos cada grupo nos
pratos da balana.
Na segunda pesagem escolha as 64 moedas do prato mais leve, caso fiquem desequilibrados,
ou qualquer grupo se ficaram em equilbrio e separe em dois grupos de 32 moedas em cada prato da
balana.
Para realizar a terceira pesagem, proceda como na segunda, separando agora 16 moedas em
cada prato da balana.
Na quarta pesagem escolha um dos grupos de 16 e divida-o em dois grupos de 8 moedas para
cada prato da balana.
Chegando na quinta pesagem divida as oito moedas obtidas na pesagem anterior e divida em
dois grupos de quatro moedas para cada prato.
Agora na sexta pesagem divida um dos grupos de 4 moedas em dois grupos de 2 moedas para
cada prato.
Na stima, e ltima pesagem, escolha um grupo de 2 moedas da sexta pesagem e coloque
uma moeda em cada prato. Se houver desequilbrio as moedas so as que esto nos pratos. Se
houver equilbrio as moedas de pesos diferentes so as outras duas separadas anteriormente.

14. Quantas vezes precisamos dobrar um papel de 1mm de espessura para que a altura da pilha
chegue da Terra Lua?

Cada dobra efetuada nos fornecer uma nova altura que corresponder ao dobro da altura
anterior. A sequncia formada ser uma PG de razo 2. Portanto teremos que encontrar um termo na
PG que seja igual distncia da Terra Lua.
Seja X a distncia, em milmetros, da Terra Lua teremos ento:

1
2 . 1

=
n
X 2 : 2
n
X =
n
X 2 . 2 =
n
X 2 log ) . 2 ( log =

2 log . log 2 log n X = +
2 log
log 2 log X
n
+
=
2 log
log
1
X
n + =


15. Descubra o erro da prova da afirmao abaixo:
Afirmao: Trs igual a dois.
Seja x um nmero diferente de zero. Temos que:
3x 3x = 2x 2x
Colocando x x em evidncia, temos que:
3(x x) = 2(x x).
Cancelando x x em ambos os lados, obtemos que 3 = 2.

O erro est no cancelamento de x x. Nessa passagem efetuada uma diviso por zero.

Captulo 2 Solues dos Exerccios

1. Observe as multiplicaes a seguir:
(a) 12.345.679 x 18 = 222.222.222
(b) 12.345.679 x 27 = 333.333.333
(c) 12.345.679 x 54 = 666.666.666
Para obter 999.999.999 devemos multiplicar 12.345.679 por quanto?
Soluo:
O algarismo que desejo repetir resulta da multiplicao do prprio por 9, ou seja:
12.345.679 x 9.n = nnn.nnn.nnn (com 0 < n < 10, sendo n um nmero natural)
Como desejo repetir o algarismo 9, temos que multiplicar por 81.

2. Outro dia ganhei 250 reais, incluindo o pagamento de horas.extras. O salrio (sem horas extras)
excede em 200 reais o que recebi pelas horas extras. Qual o meu salrio sem horas extras?
Soluo:

Denominando S = salrio e HE = hora extra, teremos:
(1) S + HE = 250
(2) S = HE + 200
Substituindo a relao (2) em (1), teremos:
200 + HE + HE = 250
HE = 25
S = 225

O meu salrio sem horas extras de R$225,00.

3. Uma torneira A enche sozinha um tanque em 10 h, uma torneira B enche o mesmo tanque sozinha
em 15 h. Em quantas horas as duas torneiras juntas enchero o tanque?

Soluo: Temos que:

T = 6 horas

4. O dobro de um nmero, mais a sua tera parte, mais a sua quarta parte somam 31. Determine o
nmero.

Soluo:
Denominando de x o nmero a ser descoberto:



24x + 4x + 3x = 372

31x = 372

x = 12
5. Uma certa importncia deve ser dividida entre 10 pessoas em partes iguais. Se a partilha fosse feita
somente entre 8 dessas pessoas, cada uma destas receberia R$5.000,00 a mais. Calcule a
importncia.

Soluo:
Denominando:
P = parte que cada pessoa ir receber;
T = total a ser dividido.



4T + 200.000 = 5T

T = 200.000

A importncia de R$200.000,00.

6. Roberto disse a Valria: Pense um nmero, dobre esse nmero, some 12 ao resultado, divida o
novo resultado por 2. Quanto deu? Valria disse 15 ao Roberto, que imediatamente revelou o
nmero original que Valria havia pensado. Calcule esse nmero.

Soluo: Seja x o nmero pensado por Valria, Temos que:
1) Passo: 2x
2) Passo: 2x + 12
3) Passo:


4) Passo: x + 6 = 15
Logo; x = 9
7. Por 2/3 de um lote de peas iguais, um comerciante pagou R$8.000,00 a mais do que pagaria pelos
2/5 do mesmo lote. Qual o preo do lote todo?
Soluo:
Denominando de x o valor total do lote, teremos:


10x 120.000 = 6x
x = 30.000
O valor total do lote de R$30.000,00

8. Determine um nmero real a para que as expresses 3a+6/8 e 2a+10/6 sejam iguais.
Soluo:



9a +18 = 8a + 40

a = 22


9. Se voc multiplicar um nmero real x por ele mesmo e do resultado subtrair 14, voc vai obter o
quntuplo do nmero x. Qual esse nmero?

Soluo:

Temos:



. *+


10. Eu tenho o dobro da idade que tu tinhas quando eu tinha a tua idade. Quando tu tiveres a minha
idade, a soma das nossas idades ser de 45 anos. Quais so as nossas idades?

Soluo:

Denominando:
Minha idade = y
Sua idade = x
A diferena entre as idades = k
Teremos ento:




(1) y = 2.(x k)
(2) x + k + y + k = 45
Substituindo (1) em (2)
3x = 45
x = 15
Como y = 2x 2k e y x = k, teremos
y = k + x
y = k + 15
k + 15 = 30 2k
k = 5
y = 5 + 15 = 20

Nossas idades so: eu tenho 20 anos e voc 15 anos.

11. Um homem gastou tudo o que tinha no bolso em trs lojas. Em cada uma gastou 1 real a mais do
que a metade do que tinha ao entrar. Quanto o homem tinha ao entrar na primeira loja?

Soluo:
Denominando de x o valor com o qual o homem tenha entrado no local.
Resolvendo o problema de trs para frente, teremos:
3 local: x/2 1 = 0, x = 2, logo, ele entrou com R$2,00.
2 local: x/2 1 = 2, x = 6, logo, ele entrou com R$6,00.
1 local: x/2 1 = 6, x = 14, logo, ele entrou com R$14,00.

Ele entrou na primeira loja com R$14,00.





12. Com os algarismos x, y e z formam-se os nmeros de dois algarismos xy e yx, cuja soma o
nmero de trs algarismos zxz. Quanto valem x, y e z?

Soluo:

Os algarismos so xy e yx, e a soma zxz. Considerando os algarismos como x, y, z Temos que (10x
+ y) + (10y + x) = 100z + 10x + z e 11x + 11y = 101z + 10x (ou ainda x + 11y = 101z)

O nmero da forma zxz precisa estar entre 101 e 98 + 89 = 187, ou seja 101 < zxz <181.
Como o nmero 11x + 11y = 11(x + y), temos que o nmero um mltiplo de 11. Ou seja, as
solues possveis so os mltiplos de 11 entre 101 e 181, ou seja: 110, 121, 132, 143, 154, 165, 176.
Dentre esses, apenas o 121 tem o algarismo da centena igual ao da unidade.

Logo z = 1, x = 2. Precisamos encontrar y. Ora, x + 11y = 101z, como j enunciamos.

Fazendo as substituies: 2 + 11y = 101 => 11y = 99 => y = 9

Os algarismos so x = 2, y = 9, z = 1


13. Quantos so os nmeros inteiros de 2 algarismos que so iguais ao dobro do produto de seus
algarismos?

Soluo:

Assumindo que os algarismos que compe o nmero sejam denominados de x(algarismo das
dezenas) e y(algarismo das unidades), teremos:

10x + y = 2xy

O dobro do produto dos algarismos implica que o nmero seja par, logo, o algarismo das unidades s
poder ser: 0,2,4,6 ou 8
Porm podemos excluir y = 0 pois o produto dos algarismos seria zero.
Se y = 2, temos que 10x + 2 = 4x, soluo descartada.
Se y = 4, temos que 10x + 4 = 8x, soluo descartada.
Se y = 6, temos que 10x + 6 = 12x, ento x = 3.
Se y = 8, temos que 10x + 8 = 16x, soluo descartada.

O nico nmero que atende o problema o 36.


14. Obter dois nmeros consecutivos inteiros cuja soma seja igual a 57.

Soluo:

Denominando um nmero de x e seu posterior de x + 1, teremos:

x + (x + 1) = 57
x = 28

Os nmeros que atendem a soluo do problema so 28 e 29.








15. Qual o nmero que, adicionado ao triplo do seu quadrado, vale 14?

Soluo:

Chamando o nmero de x, temos:

x + 3x
2
= 14 x = 7/3 ou x = 2.


16. O produto de um nmero positivo pela sua tera parte igual a 12. Qual esse nmero?

Soluo:

Denominando o nmero de x, teremos:



x
2
= 36

x = 6, mas como o enunciado diz que o nmero positivo, x = 6.

17. Determine dois nmeros consecutivos mpares cujo produto seja 195.

Soluo:

Denominando um nmero mpar por 2x + 1, e seu consecutivo por 2x + 3, teremos:

(2x + 1).(2x + 3) = 195
4x
2
+ 8x 192 = 0
x' = 8 ou x = 6, fazendo as devidas substituies teremos:
Ou os nmeros so 13 e 15 ou 15 e 13.

18. A diferena entre as idades de dois irmos 3 anos e o produto de suas idades 270. Qual a
idade de cada um?

Soluo:

Chamando as idades de x e y, temos duas equaes

{




y
2
+ 3y 270 = 0
P = 270
S = 3
y = 15 ou y = 18

Como x e y representam idades consideramos apenas o valor positivo. Portanto y = 15 e x = 18. Ento
as idades dos irmos so 15 e 18 anos


19. Calcule as dimenses de um retngulo de 16 cm de permetro e 15 cm
2
de rea.

Soluo:

Denominando as dimenses de x e y, temos pelo permetro e pela rea que:

(pelo permetro) 2x + 2y = 16
x + y = 8
y = x 8
(pela rea) x.y = 15
(substituindo y) x.(x 8) = 15
x
2
8x 15 = 0
x = 5 ou x = 3
Como estamos tratando de dimenses geomtricas, usaremos somente o valor positivo, logo:
x = 5 e y = 3


20. A diferena de um nmero e o seu inverso 8/3 . Qual esse nmero?

Soluo:

Denominando o nmero de x, teremos:




3x
2
3 = 8x
3x
2
8x 3 = 0
x' = 3 ou x = 1/3



Questo 21: A soma de dois nmeros 12 e a soma de seus quadrados 74. Determine os dois
nmeros.

Soluo:

Chamando os dois nmeros de x e y temos:

{




Da primeira equao, temos que:

(x + y)
2
= (12)
2

x
2
+ y
2
+ 2xy = 144
(substituindo pela segunda equao) 74 + 2xy = 144
2xy = 70
xy = 35
x = 35/y
(substituindo na primeira equao)


35 + y
2
= 12y
y
2
12y + 35 = 0
y = 5 ou y = 7
Se y = 5, x = 7 ou se y = 7, x = 5


Os nmeros so 7 e 5.





22. Um pai tinha 30 anos quando seu filho nasceu. Se multiplicarmos as idades que possuem hoje,
obtm-se um produto que igual a trs vezes o quadrado da idade do filho. Quais so as suas idades?

Soluo:

Denominando de P = idade do pai, e F = idade do filho, temos que:

P F = 30
PF = 3F
2
P = 3F
(substituindo na primeira equao) 3F F = 30
F = 15 e P = 45

O pai tem 45 anos enquanto que o filho tem 15 anos.



23. Os elefantes de um zoolgico esto de dieta juntos. Num perodo de 10 dias devem comer uma
quantidade de cenouras igual ao quadrado da quantidade que um coelho come em 30 dias. Em um dia
os elefantes e o coelho comem juntos 1.444 kg de cenoura. Quantos quilos de cenoura os elefantes
comem em 1 dia?

Soluo:

Denominando de E = quantidade que os elefantes devem comer em um dia e C = quantidade que um
coelho deve comer em um dia, teremos:

{

()



10C = 14440 900C
2

90C
2
+ C 1444 = 0
C = 4 C< 0 (descartado)
E = 1440 kg

Um elefante come 1440 kg de cenoura por dia.


Questo 24: Sejam
1
e
2
as razes do polinmio ax
2
+ bx + c, com a 0. Calcule as seguintes
expresses, em funo de a, b e c:

(a)


(b)


(c)



Soluo:

a)



b) (



c) (



25. O nmero 3 a raiz da equao x
2
7x 2c = 0. Nessas condies, determine o valor do
coeficiente c.

Soluo:

Se 3 raiz, ento: p( 3) = 0
( 3)
2
7.( 3) 2.c = 0
c = 15


26. Encontre o polinmio p(x) = 2x
4
+ bx
3
+ cx
2
+ dx+ e que satisfaz a equao p(x) = p(1 - x).

Soluo:

p(x) = 2x
4
+ bx
3
+ cx
2
+ dx+ e
p(x 1) = 2(x 1)
4
+ b(x 1)
3
+ c(x 1)
2
+ d(x 1)+ e
p(x 1) = 2x
4
+ ( 8 b)x
3
+ (12 + 3b + c)x
2
+ ( 8 3b 2c d)x + (2 + b + c + d + e)
Pela identidade de polinmios, temos que: p(x) (p 1), comparando os coeficientes de mesmo grau
iremos concluir que:
b = 4, c = t (t ), d = 2 t, e

O polinmio procurado ser: p(x) = 2x
4
4x
3
+ tx
2
+ (2 t )x+ e


Questo 27: (OBM) Dois meninos jogam o seguinte jogo. O primeiro escolhe dois nmeros inteiros
diferentes de zero e o segundo monta uma equao do segundo grau usando como coeficientes os
dois nmeros escolhidos pelo primeiro jogador e 1.998, na ordem que quiser (ou seja, se o primeiro
jogador escolhe a e b o segundo jogador pode montar a equao 1.998x
2
+ ax + b = 0 ou ax
2
+ 1.998x
+ b = 0 etc.) O primeiro jogador considerado vencedor se a equao tiver duas razes racionais
diferentes. Mostre que o primeiro jogador pode ganhar sempre.

Soluo:

Inicialmente veja que, se num polinmio p(x) = a
n
x
n
+ + a
1
x + a
0
, tivermos a
n
+ a
1
+ a
0
= 0,
teremos p (1) = 0 e 1 raiz de p. Dessa forma se o primeiro jogador escolhe b = (1998 + a), 1 ser
raiz da equao do segundo grau que o seu oponente ir montar.
Se um polinmio tem coeficientes inteiros (na verdade vale para coeficientes racionais) e possui uma
raiz irracional do tipo a + b r (r no quadrado perfeito), ento a b r tambm raiz. Ou seja, as
raizes irracionais vm aos pares. No caso de uma equao de segundo grau, e coeficientes inteiros,
ambas as raizes so irracionais, ou ambas so racionais.
No nosso caso, como 1 j raiz, a outra raiz ser racional. Basta ver ento, apenas se 1 no raiz
multipla (pois queremos que as raizes sejam distintas). Para isso basta escolher a adequadamente.
Se o primeiro jogador escolher os nmeros a = n1998, e b = (n + 1) 1998, n > 2 e N, ele ganha. (1
no ser raiz mltipla e a equao ter duas raizes racionais distintas.)

Obs. possivel obter solues com a + b + 1998 = 0, por exemplo com
{a, b} = {2040, 5478} (soluo obtida com o auxlio de um computador.)


28. (OBM) Mostre que a equao x
2
+ y
2
+ z
2
= 3xyz tem infinitas solues onde x;y;z so nmeros
inteiros.

Soluo:

Escolhemos uma soluo para a equao. Tomamos por exemplo o terno (1,1,1)

x + 1 + 1 = 3.x.1.1 ou seja, x=1 ou x= 2

Ento, alm de (1,1,1), o terno (2,1,1) tambm soluo da equao.

Considerando agora o terno (2,1,1) teremos ento por exemplo que 2 + 1 + z =
3.2.1.z, ou seja z=1 ou z=5.

Ento alm de (2,1,1), o terno (2,1,5) tambm soluo.
De modo geral escolhendo uma terna e escolhendo um dos trs valores, sempre acharemos outro
valor que satisfaz tambm a equao. Ou seja de um terno (x
0
, y
0
, z
0
) sempre obteremos um
outro terno (x
1
, y
1
, z
1
).

Escolhendo x
0
teremos:

x + y
0
+ z
0
= 3.x. y
0
.z
0

x - 3.x. y
0
.z
0
+ y
0
+ z
0
= 0

Com essa equao do 2 grau, sabemos ento que a soma das razes dada por s = 3. y
0
.z
0


Ento x
1
> x
0
se x
0
< 3/2 . y
0
.z
0
e caso contrrio x
0
> 3/2 . y
0
.z
0
.
Com isso garantimos que a segunda raiz sempre ser maior que a primeira que foi substituda na
equao, e consequentemente, existir infinitas solues para a equao.


29. (Gazeta Matemtica, Romnia) Considere a equao a
2
x
2
(b
2
- 2ac)x + c
2
= 0; onde a;b e c so
nmeros inteiros positivos. Se n tal que p(n) = 0, mostre que n um quadrado perfeito.

Soluo:

Temos que:
0 ) ( = n p 0 ) 2 (
2 2 2 2
= + c n c a b n a
Observe que essa uma equao do segundo grau na incgnita n . Para resolver essa equao
encontremos o seu discriminante:
) 4 ( 4 4 4 4 4 ] ) 2 ( [
2 2 2 4 2 2 2 2 2 4 2 2 2 2
ac b b c ab b c a c a c ab b c a ac b = = + = = A
Da, podemos concluir que ) 4 (
2 2
ac b b um quadrado perfeito, pois N ne raiz de ) (x p , ou seja,
0 ) ( = n p e a , b e c so inteiros positivos o que nos garante que ac b 4
2
tambm um quadrado
perfeito, j que
2
b tambm o . Mas ac b 4
2
o discriminante da equao do segundo grau
0
2
= + + c x b x a , onde suas razes
a
b
x
2
'
A +
= e
a
b
x
2
"
A
= , devido ao valor de A, so
nmeros racionais.
Voltando para a resoluo da equao inicial, 0 ) 2 (
2 2 2 2
= + c n c a b n a , temos que:
a
c
a
c a b b
a
b
a
c a b b c a b
a
c a b b c a b
n

=

=

=
2
4
2
4 2
2
) 4 ( 2
2
2
2 2
2
2 2 2

Mas,
" ' x x
a
b
+ = , " ' x x
a
c
= , '
2
4
2
x
a
c a b b
=
+
e "
2
4
2
x
a
c a b b
=

, onde ' x e " x so, como j
vimos, as razes da equao 0
2
= + + c x b x a .
Portanto, fazendo as devidas substituies, temos que:
" ' " ' ) ' ( " ' ' ) " ' ( '
2
x x x x x x x x x x n + = + =
2
) ' ( ' x n =
Ou:
" ' " ' ) " ( " ' " ) " ' ( "
2
x x x x x x x x x x n + = + =
2
) " ( " x n =
Logo, como n um nmero natural que igual ao quadrado de nmeros racionais, ento esses
nmeros racionais tem que ser inteiros e n , portanto, um quadrado perfeito.


Questo 30: (Gazeta Matemtica, Romnia) Sejam a, b Z. Sabendo que a equao
(ax b)
2
+ (bx a)
2
= x, tem uma raiz inteira, encontre os valores de suas razes.

Soluo:

(ax b)
2
+ (bx a)
2
= x

a
2
x
2
2abx + b
2
+ b
2
x
2
2abx + a
2
= x

(a
2
+ b
2
)x
2
(4ab + 1)x + a
2
+ b
2
= 0

Se o produto igual a 1 e uma as razes inteira, ento elas so inversas e:



4ab + 1 2.(a
2
+ b
2
)

2a
2
+ 2b
2
4ab 1

2.(a b)
2
1






31. (Gazeta Matemtica, Romnia) Resolva a equao:

*

+

Obs.: [x] o menor inteiro maior ou igual a x.

Soluo:

Por definio de [a], teremos que: x , 2x
2
e x
2
+ 1 .
A expresso acima equivale a dizer que x o quociente da diviso euclidiana de 2x
2
por x
2
+ 1, isto ,
equivale a dizer que: r {0, 1, ..., x, ..., x
2
}, tal que:
2x
2
= (x
2
+ 1).x + r, da podemos afirmar

x
3
+ 2x
2
x = r
x.( x
2
+ 2x 1) = r

mas temos que x | r, ento conclumos que r {0, x, 2x, ..., x
2
}, o que nos permite dizer que r = .x
com {0, 1, ..., x}. Segue que

x.( x
2
+ 2x 1) = .r
x.( x
2
+ 2x (1+)) = 0
Segue da que x = 0 ou que ( x
2
+ 2x (1+)) = 0. Da segunda igualdade, termos que ,
mas como x e {0, 1, ..., x}, = 0, e da, termos x = 1 como a segunda soluo.

32. Demonstrar que:
(a) n
4
+ 4 no primo se n > 1;
(b) generalize, mostrando que n
4
+ 4n no primo, para todo n > 1.


a)

no primo se n > 1

Soluo:
Considere a igualdade de Sophie Germain
a
4
+ 4b
4
= (a
2
+ 2b
2
+ 2ab)( a
2
+ 2b
2
- 2ab)
Tomando a = n e b = 1, temos:
n
4
+ 4 = (n
2
+ 2 + 2n)( n
2
+ 2 - 2n), assim, n
4
+ 4 composto.

b) Generalize, mostrando que n
4
+ 4
n
no primo, para todo n > 1.
Soluo:
Se n for par, segue que n
4
e 4
n
so pares, logo n
4
+ 4
n
no primo.
Se n for mpar, n da forma 2m + 1, ento, substituindo temos:
(2m + 1)
4
+ 4
2m + 1

(2m + 1)
4
+ 4 - 4
2m

(2m + 1)
4
+ 4 - 2
4m

(2m + 1)
4
+ 4 -


Tomando a = 2m + 1 e b = 2
m
, o resultado conseqncia da igualdade de Sophie Germain.



Questo 33: Para fazer 12 bolinhos, preciso exatamente de 100 g de acar, 50 g de manteiga, meio
litro de leite e 400 g de farinha. Qual a maior quantidade desses bolinhos que serei capaz de fazer com
500 g de acar, 300 g de manteiga, 4 litros de leite e 5 kg de farinha?

Soluo:

Para fazer 12 bolinhos:
100g de Acar
50g de Manteiga
0,5 l de Leite
400g de Farinha.

Com:
500g de Aucar conseguimos fazer 60 bolinhos
50g de Manteiga conseguimos fazer 72 bolinhos
0,5l de Leite conseguimos fazer 96 bolinhos
400g de Farinha conseguimos fazer 150 bolinhos
Portanto o acar o ingrediente limite e podemos fazer 60 bolinhos no
mximo.


34. Dadas as fraes



qual maior?

Soluo:

Denominando x = , x +1 = e x + 2 = , teremos uma
expresso a qual iremos supor uma afirmao, de que a primeira frao menor do que a segunda.




O que verdade, logo, a primeira frao menor do que a segunda!


35. Achar o maior valor inteiro positivo de n tal que n
200
< 5
300
:

Soluo:

Aplicando as propriedades das potncias, temos que:
(


Como a base positiva , segue que:
n
2
< 5
3

n <
n < 11,2

Como n , temos que o maior valor inteiro positivo ser quando n = 11.


36. Achar o menor valor inteiro positivo de n tal que




Soluo:



1 + 2 + 3 + ... + n > 55

( )



n
2
+ n 110 > 0

n > 0 (positivo), logo, n = 11.



37. Nove cpias de certas notas custam menos de R$ 10,00 e dez cpias das mesmas notas (com o
mesmo preo) custam mais de R$ 11,00. Quanto custa uma cpia das notas?

Soluo:

Denominando de x = valor de uma cpia, teremos que:

9x < 10 x < 10/9
10x > 11 x > 11/10
Analisando as desigualdades, x < 1,111... e x > 1,10, iremos concluir que x = 1,11.

Uma cpia custa R$1,11.


38. Se enumeram de 1 at n as pginas de um livro. Ao somar estes nmeros, por engano um deles
somado duas vezes, obtendo-se o resultado incorreto: 1.986. Qual o nmero da pgina que foi
somado duas vezes?


Soluo:

Considere k o nmero que foi somado duas vezes. O valor de k um nmero natural maior ou igual a
1 e menor ou igual a n.


Assim:
i) 1986 (1 + 2 + ... + n) 1
1986 [
()

- 1
n
2
n 3970
n
2
+ n 3970 0
Resolvendo a inequao, encontramos:
- 63,5 n 62,5
ii) 1986 (1 + 2 + ... + n) n
1986 [
()

- n
n
2
n 3972
n
2
+ 3n 3972 0
Resolvendo a inequao, encontramos:
n - 64,5 e n 61,5
As duas inequaes apresentam o seguinte conjunto soluo para n natural
61,5 n 62,5,
Segue que o nico valor de compreendido nesse intervalo 62.
Assim, o nmero de pginas do livro 62 pginas
A soma das pginas seria:
1 + 2 + 3 + ... + 62 =
()

.
Como havia somado 1986, logo: 1986 1953 = 33.

A pgina somada duas vezes, foi a pgina 33.


39. Determine os valores de a para os quais a funo quadrtica ax
2
ax + 12 sempre positiva.

Soluo:

Para que a funo seja sempre positiva deve obedecer:
a > 0 e < 0
= a
2
4.a.12 < 0 0 < a < 48.

Para uma equao ter apenas resultados positivos necessrio que delta seja
menor que zero, e o coeficiente do termo quadrtico positivo.


40. Ache os valores de x para os quais cada uma das seguintes expresses positiva:

()

()


()




a) Soluo:

Pelo numerador temos que teremos uma imagem positiva para x > 0 e imagem negativa para x < 0.
Enquanto que para o denominador teremos uma imagem positiva para qualquer valor de x
(discriminante menor que zero)
Fazendo o estudo dos sinais, veremos que o valor que satisfar a condio do problema ser:
S = { x / x > 0}


b) Soluo:

Pelo numerador temos que teremos uma imagem positiva para x > 3 e imagem negativa para x < 3.
Enquanto que para o denominador teremos uma imagem positiva para x > 1 e teremos uma imagem
negativa para x < 1.
Fazendo o estudo dos sinais, veremos que o valor que satisfar a condio do problema ser:

S = { x / x < 1 ou x > 3}


c) Soluo:

Pelo numerador temos que teremos uma imagem positiva para x < 1 ou x > 1 e imagem negativa
para 1 < x < 1.
Enquanto que para o denominador teremos uma imagem positiva para x < 0 ou x > 3 e teremos uma
imagem negativa para 0 < x < 3.
Fazendo o estudo dos sinais, veremos que o valor que satisfar a condio do problema ser:

S = { x / x < 1 ou 0 < x < 1 ou x > 3}


41. Resolve r a equao:

[x]{x}+ x = 2{x}+ 10,

onde [x] denota a parte inteira de x. Por exemplo, [2;46] = 2 e [5;83] = 5. O nmero {x} chamado
parte fracionria de x e definido por {x} = x - [x].


Soluo:
Considere x = [x] + {x} e substituindo na equao, temos:
|x|{x} + [x] + {x} = 2{x} + 10
|x|{x} + [x] {x} = 10
{x} ( [x] 1) = 10 [x]
{x} =
,-
,-
, da expresso temos que considerar que:
i) 0 < {x} 1, visto que a parte fracionria tem que ser um valor entre 0 e 1.
ii) [x] 10, pelo fato de que, se fosse maior que 10, teramos uma parte fracionria negativa.
iii) 10 [x] [x] 1, isto , o numerador tem que ser menor ou igual ao denominador.
Resolvendo essa inequao temos:
[x] [x] 1 10
2 [x] 11
[x] 5,5 , mas como [x] tem que ser inteiro, ento [x] 6
Assim, 6 [x] 10.
Substituindo em {x} =
,-
,-
, obtemos:
i) Para [x] = 6 , {x} =

iv) Para [x] = 9 , {x} =


ii) Para [x] = 7 , {x} =

v) Para [x] = 10 , {x} =


iii) Para [x] = 8 , {x} =



Assim, a soluo : 6, 8; 7,5;

; 9,125 e 10

42. Mostre que entre os retngulos com um mesmo permetro, o de maior rea um quadrado.

Soluo:
Considere um retngulo cujos lados medem x e y. Temos que a rea, A, desse retngulo dada por
e seu permetro, p, dado por

.
Logo, .

.
E mais,

. Portanto, para A mximo temos x = y.



43. Entre todos os tringulos issceles com permetro p fixado, ache as dimenses dos lados daquele
que possui a maior rea.
Soluo:
Denominando cada um dos dois lados congruentes de a e o terceiro lado de b e o permetro de p,
teremos que:
p = a + a + b
p = 2a + b
Usando a frmula de Hern temos que a rea de um tringulo com permetro p dada pela expresso

/ .

/ .

/
No caso especfico do tringulo issceles, ela ficar

/ .

/ .

/
Analisando uma parte da frmula: .

/ .

/ .

/,
Temos que M
G
M
A
, logo,
.

/ .

/ .


O que nos permite concluir que


(.

/ .

/ .



(.

/ .

/ .

/ .

/ .


O maior valor da desigualdade que o primeiro membro atingir ser quando ele for igual ao segundo
membro, logo, substituindo na frmula de Hern, teremos:


Logo a maior rea possvel

, a qual atingida quando


.

/ .

/
Ou seja, quando o tringulo for eqiltero.

44. (OBM Jnior 1993)
dada uma equao do segundo grau x
2
+ ax + b = 0, com razes inteiras a
1
e b
1
. Consideramos a
equao do segundo grau x
2
+ a
1
x + b
1
= 0. Se a equao x
2
+ a
1
x + b
1
= 0 tem razes inteiras a
2
e b
2
,
consideram s a equao x
2
+ a
2
x+ b
2
= 0. Se a equao x
2
+a
2
x+b
2
= 0 tem razes inteiras a
3
e b
3
,
consideramos a equao x
2
+ a
3
x+ b
3
= 0. E assim por diante . Se encontramos uma equao com
< 0 ou com razes que no sejam inteiros, encerramos o processo. Por exemplo, se comeamos com a
equao x
2
= 0 podemos continuar o processo indefinidamente. Pede-se:
(a) Determine uma outra equao que, como x
2
= 0, nos permita continuar o processo indefinidamente;
(b) Determine todas as equaes do segundo grau completas a partir das quais possamos continuar o
processo indefinidamente.

Seja P
0
(x) = x
2
+ a
0
x + b
0
= 0 uma equao com razes inteiras a
1
e b
1
.
Vamos definir a equao P
i
(x) = x
2
+ a
i
x + b
i
= 0 como sendo a equao obtida no i-simo passo do
procedimento descrito pelo enunciado, ou seja, a
i
e b
i
so as razes inteiras da equao P
i 1
(x) = x
2
+
a
i - 1
x + b
i - 1
= 0, para i = 1, 2, ... .
Responderemos agora os dois itens.
a) Soluo:
Seja P
0
(x) = x
2
+ kx = 0, onde k .
Como se v facilmente, podemos obter a partir de P
0
(x) = 0 as equaes:
P
i
(x) = x
2
+ ( 1)
i
kx = 0, i = 1, 2, ... .

b) Soluo:
Nosso prximo passo ser determinar todas as equaes que nos possibilitem continuar o processo
indefinidamente.
Vimos no item (a) que as equaes x
2
+ kx = 0, k , tm essa propriedade, por isso suponhamos b
0

0 em P
0
(x) = x
2
+ a
0
x + b
0
= 0. Evidentemente b
i - 1
= a
i
.b
i
, ou seja, b
i
divide b
i 1
, o que mostra que,
para todo i inteiro positivo, b
i
divide b
0
.
Alm disso, b
0
possui um nmero finito de divisores, pois b
0
0.
Da resulta que, se b
0
0 e o processo continua indefinidamente, pelo menos um dos divisores b
j
de b
0

deve aparecer infinitas vezes como termo independente das equaes do processo.
Digamos que b
j
volte aparecer como termo independente de uma equao P
k
(x) = x
2
+ a
k
x + b
k

= 0 para algum k > j.
Mas como b
i
divide b
i 1
para todo i inteiro positivo, temos |b
i 1
| |b
i
| e da |b
j
| |b
j+1
| ... |b
k
| = |b
j
|, o
que mostra que |b
i
| = |b
j
| para i = j + 1, j + 2, ..., k, onde b
i
{ b
j
, b
j
} para todo i > j.
H dois casos a considerar:

,

()

()



e
,

()

()




No primeiro caso, observemos que

raiz de

() , o que implica que a outra raiz 1 e

()

.
Se o termo independente de

()

, ento 1 raiz de

(), o que implica que

()

, o que absurdo pois

divide b
0
.
Se o termo independente de

()

, ento 1 raiz de

(), o que mostra que

()

()

()

, o que uma equao cujas razes


no so inteiras.
Vemos portanto que neste caso no possvel continuar o processo.
Uma anlise inteiramente anloga a do primeiro caso permite-nos concluir, no segundo caso,

()

que uma equao onde uma das razes 1 ou 1.


Se a raiz for 1, ento

()

, o que absurdo.
Se a raiz for 1, ento

()

.
Logo

()

, cujas razes so 1 e 2.
Assim podemos fazer

()

para i j + 2, e repetir este procedimento


indefinidamente.
imediata a verificao de que

()

tambm implica que

()


para 0 i j.
Segue que

()

a nica equao com termo independente no nulo a partir da


qual podemos repetir o processo um nmero arbitrrio de vezes.










































Captulo 3 Solues dos Exerccios
1. Encontre o resto que deixa
(a) 2001. 2002 . 2003 . 2004 + 2005
2
quando dividido por 7;

O produto 2001. 2002 . 2003 . 2004 divisvel por 7 pois 2002 0 mod 7.
Como 2005 3 mod 7 temos que 2005
2
3
2
mod 7 2 mod 7.
Logo o resto ser igual a 2.

(b) 2
100
quando dividido por 3;

Sabemos que 2
2
1 mod 3 e ( )
50
2 100
2 2 = portanto 2
100
1
50
mod 3 1 mod 3.
Ento o resto ser igual a 1.

(c) ( )
28
56
34 12371 + quando dividido por 111.

Sabemos que 12371 50 mod 111; 12371
2
58 mod 111; 12371
8
58
4
mod 111 46 mod 111.
Como ( ) 111 mod 16 111 mod 46 12371 12371
7
7
8 56
= .
Portanto devemos encontrar o resto da diviso de ( )
28 28
50 34 16 = + por 111.
Temos que 111 mod 58 50
2
; como ( )
14
2 28
50 50 = ento 111 mod 58 50
14 28
.
Sendo 111 mod 34 58
2
logo ( ) 111 mod 34 58 58
7
7
2 14
= .
Se
4 3 7
34 . 34 34 = e 111 mod 10 34
3
e 111 mod 7 34
4
ento 111 mod 10 . 7 34
7
.
Portanto temos um resto igual a 70.
2. Provar que o nmero n
5
+ 4n divisvel por 5 para qualquer nmero natural n.

Adicionando e subtraindo 5n expresso n
5
+ 4n teremos ento n
5
+ 4n + 5n 5n = n
5
n + 5n.
Pelo pequeno teorema de Fermat temos que n n
5
5 e n 5 5 .
Sabemos que se b a e c a ento c b a + .
Portanto n n 4 5
5
+ .


3. Prove que se n mpar

(a) n
3
n divisvel por 24;

Seja ( ) ( ) 1 . . 1
3
+ = = n n n n n P um nmero inteiro com n mpar.
i) Sabemos que o produto de dois nmeros consecutivos mltiplo de 2, ento P divisvel por 2.

ii) Sendo n mpar temos que ( ) 1 n e ( ) 1 + n so pares ento P divisvel por 2
2
.

iii) O produto de trs nmeros consecutivos divisvel por 3, ento P divisvel por 3.

iv) Todo nmero divisvel por 2 e 3 divisvel por 6, ento P divisvel por 6.

v) Se P mltiplo de 6 de e 2
2
ento ser mltiplo de 24.


(b) n
2
1 divisvel por 8;

Seja ( )( ) 1 . 1 1
2
+ = = n n n P um nmero inteiro com n mpar.
Sendo 1 2 + = k n , para k e Z temos que ( ) ( ) ( ) ( ) ( ) ) 1 .( 4 2 2 . 2 1 1 2 . 1 1 2 1
2
+ = + = + + + = = k k k k k k n P .

i) Se k par temos que P = 4k = 8m.
ii) Se k mpar k + 1 par ento temos P = 8m.
Portanto P ser sempre mltiplo de 8.

(c) n
2
+ (n + 2)
2
+ (n + 4)
2
+ 1 divisvel por 12.

Seja ( ) ( ) 1 4 2
2 2 2
+ + + + + = n n n P um nmero inteiro com n mpar.
Desenvolvendo temos 1 16 8 4 4
2 2 2
+ + + + + + + = n n n n n P = ( ) 7 4 3 21 12 3
2 2
+ + = + + n n n n .
Sendo 1 2 + = k n , para k e Z temos que ( ) ( ) ( ) 7 1 2 4 1 2 3
2
+ + + + = k k P
( ) 7 4 8 1 4 4 3
2
+ + + + + = k k k P ( ) 12 12 4 3
2
+ + = k k P ( ) ( ) 3 3 4 3
2
+ + = k k P
( ) 3 3 12
2
+ + = k k P m P 12 = , m e Z, logo P mltiplo de 12.


4. O nmero 2
1093
2 divisvel por 1093
2
?
Como 1093 primo e ( ) 1 1093 , 2 = , temos, pelo Pequeno Teorema de Fermat, que 2 2 1093
1093
.
Sabemos que ( ) 1 2 . 2 2 2
1092 1093
= .
Da aritmtica dos restos temos que
2
mod mod p b a p b a
p p p p
, com a, b, p e N e p primo.
Fazendo p = 1093 temos que
2 1093 1093 1093 1093
1093 mod 1093 mod b a b a .
Temos que 1 2 1 1093 . 2 2187 3
7
+ = + = = p logo ( ) 1093 mod 1 1093 . 2 3
7
+
( ) p p mod 1 2 3
7
+ ( ) ( )
2 2
2
7
mod 1 2 3 p p + ( )
2 2 14
mod 1 4 4 3 p p p + +
Como
2 2
mod 0 4 p p ento ( )
2 14
mod 1 4 3 p p + (1)
11 15 11 1093 . 15 16384 2
14
= = = p
( ) p p mod 11 15 2
14
( ) ( )
2 2
2
14
mod 11 15 2 p p ( )
2 2 28
mod 121 330 225 2 p p p +
( )
2 28
mod 121 330 2 p p + multiplicando por 3
2
temos ( )
2 28 2
mod 1089 2970 2 . 3 p p + .
Como 2970p = 1093p + 1876p + p; 1089 = 1093 4;
2 2
mod 0 p p e sendo p = 1093 temos:
( )
2 28 2
mod 4 1876 2 . 3 p p que dividido por 4 resulta em ( )
2 26 2
mod 1 469 2 . 3 p p +
( ) ( )
2 7
7
26 2
mod 1 469 2 . 3 p p + ( )
2 7 182 14
mod 1 469 2 . 3 p p +
Desenvolvendo o binmio e excluindo as congruncias
2
mod 0 p restar:
( )
2 182 14
mod 1 3283 2 . 3 p p + ( )
2 182 14
mod 1 4 1093 . 3 2 . 3 p p p + +
( )
2 2 182 14
mod 1 4 3 2 . 3 p p p + + ( )
2 182 14
mod 1 4 2 . 3 p p + (2)
Usando as expresses (1) e (2) temos
2 182
mod 1 2 p ( ) ( )
2 6
6
182
mod 1 2 p
2 1092
mod 1 2 p multiplicando por 2 temos que
2 1093
mod 2 2 p .
Logo 2 2 1093
1093
.

5. Prove que 1 999994
1234567890
divisvel por 333331.

Considere 1 999994
1234567890
= P .
Podemos escrever ( )

=
= =
1234567889
1
1234567890
999994 . 1 999994 1 999994
k
k
P .
Logo M P
k
k
. 999993 999994 . 999993
1234567889
1
= =

=
, sendo M e N.
Como 333331 . 3 999993 = ento S M P . 333331 . 333331 . 3 = = , sendo S e N.
Portanto P mltiplo de 333331.


6. O nmero N = 4
2005
+ 2005
4
primo?

( )
4
501 4 2004 4 2005 4
4 . 4 2005 4 . 4 2005 4 2005 + = + = + = N .
Fazendo 2005 = a e
501
4 = b , segue pela desigualdade de Sophie Germain que N composto.


7. Demonstre que o nmero

zeros 2006
001 ... 1000 composto.

1000...001 =

zeros 2007
00 ... 1000 + 1 = 1 10
2007
+

1 10
2007
+ = ( )
3
3
669
1 10 +

Fazendo ( )
3
3
669
1 10 = = b e a temos que ( )( )
2 2 3 3
. b b a a b a b a + + = + .
Como ( )( )
2 2 3 3
. b b a a b a b a + + = + composto ento

zeros 2006
001 ... 1000 tambm composto.
8. Utilizando o fato de que o resto de um quadrado quando dividido por 4 s pode ser 0 ou 1, d uma
outra soluo para o problema do Exemplo 3.54.

Seja P = aabb um nmero natural quadrado perfeito a e {1, 2, ..., 9} e a = b.
Temos que P = 4q + 0 ou P = 4q + 1.
Para P = 4 mod 0 aabb temos que b = 0; b = 4 ou b = 8.
Para b = 8, P no quadrado perfeito, pois nenhum quadrado perfeito termina em 8.
Para b = 0, P no quadrado perfeito, pois teramos 100 . aa P = e no existe nenhum quadrado
perfeito da forma aa.
Para b = 4, P = aa44 temos que 100 30 < < P . Por tentativa temos que 88
2
= 7744, portanto a = 7.

Para P = 4 mod 1 aabb 1 11 4 b , logo b = 3 ou b = 7. Nestes casos P tambm no quadrado
perfeito, pois nenhum quadrado perfeito termina em 3 ou em 7.


9. Dados trs inteiros, x, y, z, tais que x
2
+ y
2
= z
2
, mostre que x e y no so ambos mpares e que xy
mltiplo de 6.

I) Suponha por absurdo x e y mpares. Sendo x = 2t + 1 e y = 2v + 1 e {t, v} c Z.
Como x
2
+ y
2
= z
2
ento
2 2 2
1 4 4 1 4 4 z v v t t = + + + + + ( )
2 2 2
2 4 z v v t t = + + + + 4m + 2 = z
2
,
mostrando que z
2
par logo z tambm par.
Seja z= 2u com u e Z, ento z
2
= 4u
2
.
Portanto 4m + 2 = 4u
2
dividindo por 2 temos

par
mpar
u m
2
2 1 2 = +

o que um absurdo.
II) Mostrar que xy mltiplo de 3
Suponha x e y no mltiplos de 3. Logo z = 3k, k e Z.
Seja { } { } 2 , 1 , 3 3
2 1 2 2 1 1
c + = + = r r com r q y e r q x .
Como x
2
+ y
2
= z
2
temos que
2 2
2 2 2
2
2
2
1 1 1
2
1
9 6 9 6 9 k r r q q r r q q = + + + + + ento
2
2
2
1
3 r r + .
Como x e y no so ambos mpares e xy mltiplo de 3 ento xy mltiplo de 6.


10. Demonstre que o quadrado de um inteiro da forma 8n ou 8n+1 ou 8n + 4.

Seja m um nmero inteiro. Podemos escrever m = 4k; m = 4k + 1; m = 4k + 2 ou m = 4k + 3.
Se m = 4k ento m
2
= 16k
2
= 8(2k
2
) = 8n.
Se m = 4k + 1 ento m
2
= (4k + 1)
2
= 16k
2
+ 8k + 1 = 8 (2k
2
+ k) + 1 = 8n + 1.
Se m = 4k + 2 ento m
2
= (4k + 2)
2
= 16k
2
+ 16k + 4 = 8 (2k
2
+ 2k) + 4 = 8n + 4.
Se m = 4k + 3 ento m
2
= (4k + 3)
2
= 16k
2
+ 24k + 9 = 16k
2
+ 24k + 8 + 1 = 8(2k
2
+ 3k + 1) + 1 = 8n + 1.

Outra soluo:

Seja p = 4q + r a diviso de p por 4. Assim ( )
2 2 2 2 2
2 8 8 16 r qr q r qr q p + + = + + = .
Logo 8 mod
2 2
r p , como r e{0, 1, 2, 3} ento r
2
e {0, 1, 4}.
Portanto n p 8
2
= ou 1 8
2
+ = n p ou 4 8
2
+ = n p .


11. Trs nmeros primos p; q e r, maiores que 3, formam uma progresso aritmtica, ou seja, q = p + d
e r = p + 2d. Prove que d divisvel por 6.

Para provar que d divisvel por 6, devemos mostrar que 2|d e 3|d.
Como todos os primos maiores que 3 so mpares e, supondo que d seja mpar, temos que q par,
pois, a soma de dois nmeros mpares resulta em um nmero par, o que um absurdo, pois, q
primo. Assim, d par, isto , 2|d.
Supondo que d no seja divisvel por 3, ento d da forma 3k + 1 ou 3k + 2, com k eN.
Para d = 3k + 1, verificamos que:
p = p q = p + d = p + 3k + 1 r = p + 2(3k + 1) = p + 3(2k) + 2

Para d = 3k + 2, verificamos que:
p = p q = p + d = p + 3k + 2 r = p + 2(3k + 2) = p + 3(2k + 1) + 1

Assim, pelo Lema dos restos, na diviso por 3 os valores de q e r so da forma p + 1 e p + 2.
Logo, sendo p, q e r trs nmeros consecutivos, ou seja, p, p + 1 e p + 2, ento, pelo menos um deles
mltiplo de 3, mostrando que 3| d.
Portanto, como 2|d e 3|d, ento 6|d.

12. Demonstrar que existem infinitos nmeros primos da forma 4m + 3 e da forma 6m + 5, onde m e Z.

Um nmero inteiro P qualquer pode ser inscrito como r q P + = 4 , com r e {0, 1, 2, 3}. Se P primo
ento r = 1 ou r = 3.
Suponha que existam finitos primos na forma 4m + 3 e seja A o conjunto de todos os primos dessa
forma, ou seja { } { }
n
p p p p N m m A ,..., , 5 , 2 , 1 4
2 1 0
= = = e + = .
Observe que o conjunto A fechado multiplicativamente.
Suponhamos, por absurdo, que existam apenas um nmero finito de primos p
1
< ... < p
k
da forma
3 4 + m , com m > 1. O nmero P = 4(p
1
.p
2
....p
k
) + 3 no divisvel por nenhum dos nmeros primos 3,
p
1
, ..., p
k
e, portanto, sua decomposio em fatores primos s pode conter primos da forma 4m + 1, o
que uma contradio, pois P da forma 4m + 3.

Considere um nmero natural S qualquer. Podemos escrev-lo na forma r m+ 6 , { } 5 ,..., 1 , 0 e r .
Se o nmero S primo ento { } 5 , 1 e r .
Suponhamos, por absurdo, que existam apenas um nmero finito de primos p
1
< ... < p
k
da forma
5 6 + m , com m > 1. O nmero S = 6(p
1
.p
2
....p
k
) + 5 no divisvel por nenhum dos nmeros primos 5,
p
1
, ..., p
k
e, portanto, sua decomposio em fatores primos s pode conter primos da forma 6m + 1.
Como o conjunto B = {6n + 1; neN} fechado multiplicativamente, ento S da forma 6m + 1, o que
uma contradio. Logo existem infinitos nmeros primos da forma 6m + 5.

13. Encontrar o ltimo dgito dos nmeros (resto da diviso por 10)

(a) 1989
2005

Sabemos que ( )
1002
2 2005
1989 . 1989 1989 = .
10 mod 9 1989 e 10 mod 1 10 mod 81 10 mod 9 1989
2 2

( ) 10 mod 1 10 mod 1 1989
1002
1002
2
.
( ) 10 mod 9 10 mod 1 . 9 1989 . 1989 1989
1002
2 205
=
Logo o dgito da unidade 9.


(b) 777
777
+ 2
50

10 mod 1 81 3
4
e 10 mod 7 27
10 mod 7 777 ( ) ( ) 10 mod 7 10 mod 1 . 7 3 . 27 3 . 3 343 7 777
64
4 256 3 259
259
3 777

10 mod 2 2
5
( ) 10 mod 4 1024 2 2 2
10
10
5 50

Logo ( ) 10 mod 1 10 mod 4 7 2 777
50 777
+ + , ento o algarismo da unidade 1.


(c) 1 + 2
2
+ 3
2
+ ... + 2005
2
.

Sabemos que
10 mod 1 1991 ... 21 11 1
;
10 mod 2 1992 ... 22 12 2
, ou seja nmeros
de mesmo final possuem mesmos restos mod 10 e sabemos, tambm, que todo nmero quadrado s
termina em 0, 1, 4, 5, 6 ou 9.
Dividindo 2000 em grupos de 10 temos 200 grupos.
Portanto temos
i) de 1 at 2000 temos
( ) 10 mod 0 9000 45 . 200 0 1 4 9 6 5 6 9 4 1 . 200 = = + + + + + + + + +

ii) de 2001 at 2005 temos
10 mod 5 25 5 4 9 4 1 = + + + +

Portanto o algarismo da unidade 5.

14. Prove que a soma dos quadrados de cinco nmeros consecutivos no um quadrado perfeito.

Sejam 2 ; 1 ; ; 1 ; 2 + + n n n n n nmeros naturais consecutivos.
Somando os quadrados ( ) ( ) ( ) ( )
2 2 2 2 2
2 1 1 2 + + + + + + n n n n n temos 10 5
2
+ n .
Se n par (n = 2k) temos 2 4
'
+ k . Se n mpar (n = 2k + 1) temos 3 4
' '
+ k .
Como um quadrado perfeito da forma k 4 ou 1 4 + k .
Portanto a soma dos quadrados de cinco nmeros consecutivos no um quadrado perfeito.

15. Prove que 1 00 ... 000 5 00 ... 000 1
100 100

zeros zeros
no um cubo perfeito.

Se n inteiro podemos escrever Z e = = k k n q n , 9 3
3
ou Z e + = + = k k n q n , 1 9 1 3
3
ou
Z e + = + = k k n q n , 8 9 2 3
3
.
Seja 1 00 ... 000 5 00 ... 000 1
100 100

zeros zeros
X = ento podemos escrever 1 10 . 5 10
101 202
+ + = X .
Pelo lema dos restos temos que 9 mod 1 10
202
; 9 mod 1 . 5 10 . 5
101
e 9 mod 1 1 .
Logo X deixa resto 1 + 5 + 1 = 7 quando dividido por 9, portanto X no cubo perfeito.

16. Seja b um inteiro positivo. Enuncie e prove o critrio de divisibilidade por b no sistema de
numerao de base b.

Proposio: Se ( ) Z a a a a P
b n n
e =
0 1 1
... com { } { } n i b a
i
,..., 2 , 1 , 0 , 1 ,..., 2 , 1 , 0 e e , ento P b
0
0
= a .
() Seja P um nmero escrito na base b, logo
0 1
1
1
... a b a b a b a P
n
n
n
n
+ + + + =

.
Podemos escrever
0 1
2
1
1
) ... .( a a b a b a b P
n
n
n
n
+ + + + =

.
Pela proposio 3.3 temos que P b , logo
0 1
2
1
1
) ... .( a a b a b a b b
n
n
n
n
+ + + +

.
fato que ) ... .(
1
2
1
1
a b a b a b b
n
n
n
n
+ + +

ento b tambm deve dividir


0
a . Como 1 0
0
s s b a
temos necessariamente que 0
0
= a .

(:) Seja b a b a b a P
n
n
n
n 1
1
1
... + + + =

um nmero em base b, logo


q b a b a b a b P
n
n
n
n
. ) ... .(
1
2
1
1
= + + + =

P b

17. Prove que os nmeros

(a)
n
n
1
...
3
1
2
1
1 + + + + = o , com n > 1,

Considere p o maior nmero primo tal que p < n, ento,

( ) ( ) ( )
n p
n p n p p n p n p
n

+ + + + + +
=
... ... 3 2 1
1 ... ... 2 1 ... ... 1 1 ... 2 1 ... ... ... 4 3 1 ... ... 3 2
o

Fazendo

e
logo .

Como p no divide ( )( ) n p p ... 1 1 ... 3 2 1 + , que uma parcela de m, ento pk tambm no divide
( )( ) n p p ... 1 1 ... 3 2 1 + portanto, o
n
no inteiro.

(b)
1 2
1
...
5
1
3
1
+
+ + + =
n
n
| , com n > 0, no so inteiros.

Considere p o maior nmero primo tal que p < n.
Ento,

( ) ( ) ( ) ( ) ( ) ( )
( ) 1 2 ... ... 7 5 3
1 2 ... ... 3 ... 1 2 ... 2 2 ... 5 3 ... 1 2 ... ... 7 3 1 2 ... ... 7 5
+
+ + + + + + + + +
=
n p
n p n p p n p n p
n
|

Fazendo

( ) ( ) ( ) 1 ... ... 2 1 ... ... 1 1 ... 2 1 ... ... ... 4 3 1 ... ... 3 2 + + + + + + + = n p n p p n p n p m
( ) ( ) n p p k + = ... 1 1 ... 2 1
pk
m
n
= o
( ) ( ) ( ) ( ) ( ) ( ) 1 2 ... ... 3 ... 1 2 ... 2 2 ... 5 3 ... 1 2 ... ... 7 3 1 2 ... ... 7 5 + + + + + + + + + = n p n p p n p n p m

e ( )( ) ( ) 1 2 ... 2 2 ... 7 5 3 + + = n p p k ento
pk
m
n
= | .
Como p no divide ( )( ) ( ) 1 2 ... 2 2 ... 7 5 3 + + n p p , que uma parcela de m, ento pk tambm
no divide ( )( ) ( ) 1 2 ... 2 2 ... 7 5 3 + + n p p , portanto, |
n
no inteiro.


18. Considere o polinmio
0
1
1
... ) ( a n a n a n p
m
m
m
m
+ + + =

de grau m > 1 com coeficientes inteiros e


neN. Prove que p(n) um nmero composto para infinitos valores de n.

Sugesto: Use o fato de que existe a eN tal que 1 ) ( > = a p o e mostre que o divide ( ) a k p + o , para
todo k e Z.

Sabemos que ) (a p = o e temos que
0
1
1
... ) ( a a a a a a p
m
m
m
m
+ + + =

.
Calculando ( ) a k p + o em
0
1
1
... ) ( a n a n a n p
m
m
m
m
+ + + =

temos:

0
1
1
... ) ( ) ( ) ( a a k a a k a a k p
m
m
m
m
+ + + + + = +

o o o que, aps desenvolver os binmios de cada


parcela, restar ( ) ( ) | | k k k a a a a a a k p
m m m
m
m
m
o o o o + + + + + + + = +

... ... ) (
1
0
1
1
.
Como a eN, a primeira parcela representa o = ) (a p e como k e Z, a segunda parcela representa
uma soma de potncias de o que pode ser escrita como o . Q , pois mltiplo de o.
Ento ( ) o o o o o . 1 . ) ( T Q Q a k p = + = + = +


19. Dizemos que um conjunto A
n
formado por n inteiros positivos escritos no sistema binrio (base 2)
regular se, para qualquer s inteiro no negativo a quantidade de nmeros de A
n
que contemplam 2
s
na
representao binria par. Dizemos que A
n
irregular se, pelo menos para algum s, este nmero
mpar. Demonstre que um sistema irregular pode se converter em regular excluindo-se apenas um
nico elemento do mesmo, e, um sistema regular pode se converter em irregular excluindo-se qualquer
um dos seus elementos.


20. Seja n um inteiro positivo. Demonstrar que todos os coeficientes do desenvolvimento do binmio
de Newton (a + b)
n
so mpares se, e somente se, n da forma 1 2
s
.
21. Prove que se (x
0
, y
0
) uma soluo da equao diofantina linear ax by = 1, ento a rea do
tringulo cujos vrtices so (0, 0), (b, a) e (x
0
, y
0
)
2
1
.
Se (x
0
, y
0
) soluo da equao ax by = 1, ento ax
0
by
0
= 1.
Sabemos que a rea de uma regio triangular, no plano, pode ser dada por D A .
2
1
= , sendo D o
determinante formado com as coordenadas dos vrtices.

Ento
1
1
1 0 0
2
1
0 0
a b
y x A = =
0 0
2
1
by ax = 1
2
1
=
2
1

22. Qual a menor distncia possvel entre dois pontos (x
1
, y
1
) e (x
2
, y
2
), com coordenadas inteiras,
situados sobre a reta definida pela equao diofantina ax + by = c?
Temos que c by ax c by ax = + = +
2 2 1 1
e
Ento
2 2 1 1
by ax by ax + = + logo ( ) ( )
2 1 2 1
x x a y y b = ( )
2 1 2 1
x x
b
a
y y = .
Como a distncia entre dois pontos no plano dada por ( ) ( )
2
2 1
2
2 1
y y x x d + = temos, ento que
( ) ( )
2
2 1
2
2 1
(

+ = x x
b
a
x x d ( ) ( )
2
2 1
2
2
2
2 1
x x
b
a
x x d + =
( )
|
|
.
|

\
|
+ =
2
2
2
2 1
1
b
a
x x d
|
|
.
|

\
| +
=
2
2 2
2 1
b
b a
x x d .

Se (x
1
, y
1
) = (x
2
, y
2
) e as coordenadas so inteiras temos d = 0 e 1
2 1
> x x .
A distncia mnima ocorre quando 1
2 1
= x x ento
|
|
.
|

\
| +
=
2
2 2
min
b
b a
d .















Captulo 4- Solues dos Exerccios


1. Seja C um conjunto formado por cinco pontos de coordenadas inteiras no plano. Prove que o ponto
mdio de algum dos segmentos com extremos em C tem tambm coordenadas inteiras.

Soluo:

Seja C = { P
1
, P
2
, P
3
, P
4
, P
5
} o conjuntos dos 5 pontos de coordenadas inteiras no plano.
Temos as seguintes possibilidades:

P
1
= { par, par}; P
2
= { par mpar}; P
3
= {mpar, par} e P
4
= {mpar, mpar}.

O ponto mdio entre dois pontos A e B dado por M( x
M
, y
M
) , onde:



Observe que para o ponto mdio tenha coordenadas inteiras x
A
e x
B
, necessrio que sejam ambos
pares ou ambos mpares, o mesmo ocorre com y
A
e y
B
.
Dessa forma, os pontos P
1
, P
2
, P
3
, P
4
, P
5
representam os pombos, e a paridade de suas coordenadas
representam as casas, onde no pode haver outra possibilidade.
Pelo P.C.P., P
5
ter a paridade de suas coordenadas igual a um dos pontos P
1
, P
2
, P
3
, P
4
.
Portanto, o ponto mdio de algum dos segmentos com extremidades em C tem tambm coordenadas
inteiras.


2. O conjunto dos dgitos 1, 2, ..., 9 dividido em trs grupos. Prove que o produto de alguns dos
grupos deve ser maior que 71.

Soluo:

Sejam 1, 2, 3, os produtos dos elementos do grupo 1, do grupo 2 e do grupo 3, respectivamente.
Suponha que:

1 71
2 71
3 71

Da, como 1 > 0, 2 > 0, 3 > 0, temos:

362880 = 1 2 3 357911

ou seja,

362880 357911 (absurdo!!!)

Portanto, o produto de alguns dos grupos deve ser maior que 71.




3. Prove que se N mpar ento para qualquer bijeo

p : I
N
I
N


do conjunto I
N
= {1, 2, ... ,N} o produto P(p) = (1 p(1))(2 p(2)) ... (N p(N)) necessariamente par.
(Dica: O produto de vrios fatores par se, e somente se, um dos fatores par.)

Soluo:

Se N mpar ento da forma N = 2k + 1. Se P(p) fosse mpar, todos os termos (i p(i)) seriam
mpares para todo 1 i N. Sendo assim (N p(N)) = (2k + 1 p(2k + 1)) =
2k + 1 p(2k + 1) = 2(k k), o que implica que P(N) par, logo P(p) no mpar.


4. Dado um conjunto de 25 pontos no plano tais que entre quaisquer 3 deles existe um par com
distncia menor que 1. Prove que existe um crculo de raio 1 que contm pelo menos 13 dos 25 pontos
dados.

Soluo:

(i)

Inicialmente escolhemos dois pontos P e Q do plano cuja distncia de P a Q maior
que 2, traamos dois crculos C1 e C2 de raio 1 e centros P e Q respectivamente.
Se todos os pontos estiverem no interior de um dos dois crculos o problema est resolvido. Caso
contrrio teramos um dos pontos exterior aos dois crculos o que contraria o fato de que entre
quaisquer 3 deles um par existe um par com distncia menor que 1.

(ii)

Vamos analisar a situao os crculos C1 e C2 possuem interseco no vazia, ou seja, C1 e C2
possuem interseco possui d pontos, com isso, cada crculo tem pelo menos 23 d pontos. Assim,
cada regio que no interseco entre os dois crculos tem na pior das hipteses

com 0 d 23. Se d = 0, temos a situao (i), caso contrrio, um dos crculos ter pelo
menos 13 pontos.
.

5. Prove que entre quaisquer 5 pontos escolhidos dentro de um tringulo eqiltero de lado 1 sempre
existe um par destes cuja distncia no maior que 0,5.

Soluo:

Dividindo o tringulo equiltero de lado 1 em quatro tringulos eqilteros de lado 1/2 como a figura
que se segue.









Logo, pelo PCP pelo menos dois pontos devem estar no mesmo tringulo, uma vez que temos 4
tringulos e 5 pontos.


Assim, como a maior distncia entre dois pontos que esto num tringulo equiltero ocorre quando
estes estiverem no vrtice do tringulo e como os 4 tringulos menores tem lados , ento pelo menos
dois de seus pontos esto a uma distncia de no mximo .
.

6. Marquemos todos os centros dos 64 quadrinhos de um tabuleiro de xadrez de 8x8. possvel cortar
o tabuleiro com 13 linhas retas que no passem pelos pontos marcados e de forma tal que cada
pedao de recorte do tabuleiro tenha no mximo um ponto marcado?

Soluo:

Temos marcados no tabuleiro 64 pontos, estando alinhados em 8 linhas e 8 colunas. Para separarmos
os 8 pontos pertencentes as 8 colunas so necessrias 7 linhas e para dividirmos os 8 pontos
pertencentes as 8 linhas so necessrias 7 linhas, totalizando 14 linhas. Caso sejam usadas 13 linhas
poderemos dividir o tabuleiro em 56 espaos, como temos 64 pontos, pelo Princpio da Casa dos
Pombos, teremos pelo menos 8 casas com 2 pontos. Logo, no possvel cortar o tabuleiro com 13
linhas retas que no passem pelos pontos marcados e de forma tal que cada pedao de recorte do
tabuleiro tenha no mximo um ponto marcado.

7. Prove que existem duas potncias de 3 cuja diferena divisvel por 1997.

Soluo:

Existem 1997 possveis restos pela diviso por 1997. Considere a sequncia das potncias de 3: 3
0
,
3, 3, 3, ...,3
1997
. Esta sequncia composta de 1998 nmeros.
Portanto, pelo PCP, dois desses, digamos 3n e 3m, com n > m, tm mesmo resto quando divididos por
1997. Logo, a sua diferena 3
n
3
m
divisvel por 1997.

8. So escolhidos 6 nmeros quaisquer pertencentes ao conjunto

A = {1, 2, ...., 10}.

Prove que existem dois destes seis nmeros cuja a soma mpar.

Soluo:

Sabemos que a soma de dois nmeros naturais mpar se, os nmeros no tiverem a mesma
paridade, isto , se um for par e o outro for mpar.
Como o conjunto A tem 5 nmeros pares e 5 nmeros mpares e devemos escolher 6 nmeros
quaisquer, pelo PCP, teremos dentre os seis nmeros escolhidos, pelo menos um nmero par e um
nmero mpar escolhido, onde a soma mpar.




9. Seja x um nmero real arbitrrio. Prove que entre os nmeros

x, 2x, 3x, ..., 101x

existe um tal que sua diferena com certo nmero inteiro menor 0,011.


Soluo:


Seja x um nmero real tal que, para certo inteiro se tenha a.x [, +1], para a {1, 2, ...,
101} e x

.
Considere os conjuntos I = {x
0
=
1
, x
1
, x
2
, ..., x
99
, x
100
= +1} com x
1
< x
2
< ...< x
99
<
x
100
, e x
i
x
i 1
= 0,01, i {1, 2, ..., 100}.

Considere os intervalos I
i
= [x
i 1
, x
i
] e x = ou 101.x = + 1, nada h a demonstrar.

Suponha que x e 101.x + 1.

Temos # { I
i
} = 100 e # { x, 2x, 3x, ..., 101x} = 101. Logo, temos 101 elementos para acomodar em 100
intervalos de comprimento 0,01. Segue que teremos dois nmeros em um nico intervalo. Se x > 0,
sero 100x e 101x. Se x < 0, sero x e 2x. Em qualquer caso teremos dois nmeros num mesmo
intervalo, no qual um dos extremos inteiro. Logo devemos ter x < 0,01 < 0,011 ou + 1 101x
< 0,01 < 0,011.




10. Mostre que entre nove nmeros que no possuem divisores primos maiores que cinco, existem
dois cujo produto um quadrado.

Soluo:

Seja A= {x
1
, x
2
, x
3
, x
4
, x
5
, x
6
, x
7
, x
8
, x
9
} o conjunto de nove nmeros que no possuem divisores primos
maiores que cinco. Os nmeros que no possuem divisores primos maiores que cinco so da forma x
I

= 2

.3

.5

.
Para que o produto de dois nmeros x
i
.x
j
seja um quadrado, devemos ter x
I
= 2

.3

.5

e x
j
= 2

.3

.5

,
com i,j {1, 2, 3, 4, 5, 6, 7, 8, 9}, ou seja, x
i
.x
j
= (2

.3

.5

).( 2

.3

.5

) = 2
+
.3
+
.5
+
, onde + ,
+ e + necessariamente devem ser pares, temos as seguintes possibilidades:







Possibilidades
e e e + , + , +
1 Pares Pares Pares Pares
2 Pares Pares mpares Pares
3 Pares mpares Pares Pares
4 Pares mpares mpares Pares
5 mpares Pares Pares Pares
6 mpares mpares mpares Pares
7 mpares Pares mpares Pares
8 mpares mpares mpares Pares


Ento existem 8 possibilidades para que o produto seja quadrado. Pelo P.C.P., as 8 possibilidades so
as casas e os 9 elementos de A so os pombos, e consequentemente, dentre os 9 elementos de A, o
produto de pelo menos dois deles um quadrado.


11. Um disco fechado de raio um contm sete pontos, cujas distncias entre quaisquer dois deles
maior ou igual a um. Prove queo centro do disco um destes pontos.

Soluo:

12. Na regio delimitada por um retngulo de largura quatro e altura trs so marcados seis pontos.
Prove que existe ao menos um par destes pontos cuja distncia entre eles no maior que .

Soluo:

Dividimos o retngulo em quadrados de lado unitrio, como mostra a figura abaixo. Posicionamos 4
destes pontos nos vrtices do retngulo maior. Em seguida posicionamos o quinto ponto no centro do
retngulo maior (onde do centro aos outros teremos 2,5 > ). Ao acrescentarmos o sexto ponto, em
qualquer regio, ela no ser maior que 2 em relao a 2 pontos mais prximos (lembrando que 2 <
).

13. Seja a um nmero irracional. Prove que existem infinitos nmeros racionais r = p/q tais que |a r|
j< 1/q
2
.

Soluo:

14. Suponha que cada ponto do reticulado plano pintado de vermelho ou azul. Mostre que existe
algum retngulo com vrtices no reticulado e todos da mesma cor.

Soluo:

Se imaginarmos 3 linhas horizontais, as quais possuam 3 ponto de cores alternadas (vermelho ou
azul) e alinhados sobre uma reta perpendicular s horizontais; para cada paralela ao alinhamento
desses pontos teremos 2x2x2 = 8 possibilidades de disposio de cores aos pontos de interseco
das paralelas com as horizontais. Ento, sabemos que aps 8 retas paralelas ir repetir algum padro
das 8 possibilidades, e iremos ter o possvel retngulo.



15. Um certo livreiro vende pelo menos um livro por dia. Sabendo que o livreiro vendeu 463 livros
durante 305 dias consecutivos, mostre que em algum perodo de dias consecutivos o livreiro vendeu
exatamente 144 livros.

Soluo:
A soma de livros vendidos do 1 ao ltimo dia 463. Considere S
n
como a soma dos livros vendidos do
1 dia ao dia n, com 1 305. Temos que:
1 S
1
S
2
<S
3
<S
4
< S
305
= 463.
Note S
4
- S
1
a quantidade de livros vendidos entre o 1 e o 4 dia. Considere agora 1 p
Devemos mostrar que existe p e q tal que S
p
- S
q
= 144.
Observe que a seqencia abaixo crescente, pois vendido por dia pelo menos 1 livro.
S
1
S
2
< S
3
< S
4
< S
305
(I), logo
144+S
1
144+S
2
<144+S
3
<144+S
4
< S
305
(II)
Das sequencias (I) e (II) temos:
- 305+305= 610 nmeros e
- 463+144= 607 valores possveis.
Pelo PCB, os nmeros equivalem aos pombos e os possveis valores as casas, logo S
p
seqencia (I)
e S
q
+ 144 seqencia (II).
Portanto em algum perodo de dias consecutivos o livreiro vendeu exatamente 144 livros.


















Captulo 5 Solues dos Exerccios
1. De quantas maneiras podemos escolher trs nmeros distintos do conjunto I50 = {1, 2, 3, ..., 49, 50 }
de modo que sua soma seja

a) um mltiplo de 3?

Devemos escolher nmeros cujas classes de restos so do tipo 0 0 0 ou 1 1 1 ou 0 1 2 ou
2 2 2.

Classe de resto 0 so 16 nmeros, de resto 1 ou 2 so 17 nmeros cada.

Para escolher trs nmeros de resto 0 num total de 16 temos 560
3
16
=
|
|
.
|

\
|
maneiras.

Para escolher trs nmeros de resto 1 ou 2 num total de 17 temos 1360 680 . 2
3
17
. 2 = =
|
|
.
|

\
|
maneiras.

Para escolher um nmero de resto 0, um nmero de resto 1 e um nmero de resto 2 temos, pelo
principio multiplicativo, 16 . 17 . 17 = 4624 maneiras.

Logo teremos 560 + 1360 + 4624 = 6544 maneiras distintas de escolher os trs nmeros cuja soma
ser mltiplo de trs.


b) um nmero par?
Podemos escolher trs nmeros pares num total de 25, ou seja, 2300
3
25
=
|
|
.
|

\
|
maneiras.

Podemos escolher dois nmeros mpares num total de 25, ou seja, 300
2
25
=
|
|
.
|

\
|
maneiras e um nmero
par num total de 25. Portanto teremos 300 . 25 = 7500 maneiras.

Finalmente pelo principio aditivo teremos um total de 2300 + 7500 = 9800 maneiras distintas de
escolher os trs nmeros cuja soma ser par.


2. Considere o conjunto I
n
= {1, 2, 3, ..., n 1, n}. Diga de quantos modos possvel formar
subconjuntos de k elementos nos quais no haja nmeros consecutivos?

Considerando um caso particular se o conjunto fosse {1, 2, 3, 4, 5, 6}, teramos 4 opes para
formarmos 3 subconjuntos onde no existiriam nmeros consecutivos. Seriam os seguintes
subconjuntos: {1, 3, 5} {1, 3, 6} {1, 4, 6} {2, 4, 6}
lgico que este processo de enumerao exaustivo e nada prtico, ento, vamos tentar
generalizar.
Usaremos o smbolo + para os elementos que faro parte do subconjunto e o smbolo - para os que
no faro parte dele.
Para o exemplo dado, com um conjunto de 6 elementos e subconjuntos de 3 elementos, teramos 3
smbolos + e 3 smbolos - que, em cada subconjunto no poderiam estar seguidos.
Para o subconjunto {1, 3, 5}, a simbologia seria: + - + - + -.
Devemos perceber que, para 6 elementos, ficam definidos 7 posies possveis (n + 1), conforme
figura abaixo.
- - -

Fixando os 3 lugares que seriam preenchidos pelos elementos que no faro parte do subconjunto,
sobrariam 4 posies (n k +1) para serem escolhidas 3 para serem preenchidas pelos que faro
parte do subconjunto.
Note que, se temos 3 elementos que no vo participar do subconjunto, temos 3 + 1 (n k +1)
posies para serem ocupadas pelos outros 3 elementos, que faro parte do subconjunto. Logo, em
nosso exemplo, temos uma nica posio para os no participantes (-) e C4,3 para os participantes (+)
do subconjunto.
Ento, generalizando, teremos para n elementos
k k n
C
, 1 +
.

3. Considere as letras da palavra PERMUTA. Quantos anagramas de 4 letras podem ser formados,
onde:
a) no h restries quanto ao nmero de consoantes ou vogais?
b) o anagrama comea e termina por vogal?
c) a letra R aparece?
d) a letra T aparece e o anagrama termina por vogal?

a) 7 . 6 . 5 . 4 = 840 anagramas b) 3 . 5 . 4 . 2 = 120 anagramas

c) So 4 posies para colocar a letra R. Depois temos 6 . 5 . 4 = 120 maneiras de escolher as outras
trs letras. Portanto temos 4 . 120 = 480 anagramas com a letra R.

d) So trs posies para colocar a letra T. Temos 3 . (5 . 4 . 3) = 180 anagramas com a letra T que
terminam em vogal.


4. Calcular a soma de todos os nmeros de 5 algarismos distintos formados com os algarismos 1; 3; 5;
7 e 9.

Teremos um total de 120 nmeros todos da forma e d c b a + + + + 10 . 10 . 10 . 10 .
2 3 4
. Sendo que a, b, c,
d, e podem assumir os valores 1, 3, 5, 7 e 9.
Somando esses algarismos teremos 1 + 3 + 5 + 7 + 9 = 25 (soma dos valores absolutos que
corresponde soma quando cada um deles estiver em uma ordem fixa). Como so 24 nmeros com
os algarismos ocupando uma posio fixa teremos 25 x 24 = 600.
Para somar todos os nmeros devemos usar a decomposio do total que ser dada por
) ( 10 ). ( 10 . ) ( 10 ). ( 10 ). (
2 3 4
e d c b a e d c b a e d c b a e d c b a e d c b a + + + + + + + + + + + + + + + + + + + + + + + +
multiplicado por 24. Como (a + b + c+ d + e) x 24 = 600, temos por soma o produto
( ) 1 10 10 10 10 . 600
1 2 3 4
+ + + + = 600 x 11111 = 6666600


5. Quantos nmeros podem ser formados pela multiplicao de alguns ou de todos os nmeros 2; 2; 3;
3; 3; 5; 5; 6; 8; 9; 9?

Tais nmeros so do tipo
c b a
5 . 3 . 2 , sendo 0 s a s 6; 0 s b s 8 e 0 s c s 2. Como so 7, 9 e 3
possibilidades para a, b e c, respectivamente, ento so 7 . 9 . 3 = 189 possibilidades para
c b a
5 . 3 . 2 .
Mas 1, 2, 3 , 5, 8, 3
8
, 3
8
. 5, 3
8
. 5
2
, 2
6
, 2
6
. 5 e 2
6
. 5
2
no podem ser obtidos como pede o texto.
Assim, so 178 desses nmeros.

6. Entre todos os nmeros de sete dgitos, diga quantos possuem exatamente trs dgitos 9 e os
quatro dgitos restantes todos diferentes?

Temos um total de
4 , 9
3
7
.C P nmeros de sete dgitos com trs repetidos, veja figura abaixo.
9 9 9


3
7
P
4 , 9
C
Mas, devemos excluir os nmeros iniciados em zero, que so em um total de
3 , 8
3
6
.C P

Ento teremos
4 , 9
3
7
.C P -
3 , 8
3
6
.C P = 126 x 840 56 x 120 = 99120 nmeros.


7. De quantas maneiras podemos distribuir 22 livros diferentes entre 5 alunos se 2 deles recebem 5
livros cada e os outros 3 recebem 4 livros cada?

Primeiro escolhemos os alunos de
3 , 3 2 , 5
. C C maneiras diferentes.
Depois escolhemos os livros de
4 , 4 4 , 8 4 , 12 5 , 17 5 , 22
. . . . C C C C C maneiras diferentes.
O total, pelo principio multiplicativo, ser o produto das duas escolhas.

Teremos
3 , 3 2 , 5
. C C .
4 , 4 4 , 8 4 , 12 5 , 17 5 , 22
. . . . C C C C C =
( ) ! 2 . ! 3 . ! 4 . ! 5
! 22
3
= 56463835428000


8. Quantos so os nmeros naturais de sete dgitos nos quais o dgito 4 figura exatamente 3 vezes e o
dgito 8 figura exatamente 2 vezes?

Os nmeros iniciados em 4 so iguais a 8 . 8 . . . 1
2 , 4 2 , 6
C C = 5760 (
2 , 6
C representa as maneiras de
acrescentar os outros dois dgitos 4 e
2 , 4
C representa as maneiras de acrescentar os dois dgitos 8).
Os nmeros iniciados em 8 so iguais a 8 . 8 . . . 1
3 , 5 1 , 6
C C = 3840 (
1 , 6
C representa as maneiras de
acrescentar o outro dgito 8 e
3 , 5
C representa as maneiras de acrescentar os trs dgitos 4).
Os nmeros iniciados em outros dgitos, exceto o zero, so iguais a 8 . . . 7
2 , 3 3 , 6
C C = 3360 (
3 , 6
C
representa as maneiras de acrescentar os trs dgitos 4 e
2 , 3
C representa as maneiras de acrescentar
os dois dgitos 8).
Portanto, pelo princpio aditivo, temos a soma 5760 + 3840 + 3360 = 12960 nmeros.


9. De quantas maneiras uma comisso de 4 pessoas pode ser formada, de um grupo de 6 homens e 6
mulheres, se a mesma composta de um nmero maior de homens do que de mulheres?

Se o nmero de homens maior que o nmero de mulheres ento devemos ter 3 homens e 1 mulher
ou 4 homens e 0 mulheres.
Para 3 homens e 1 mulher temos 120 .
1 , 6 3 , 6
= C C maneiras de escolher as comisses.
Para 4 homens e 0 mulheres temos 15
4 , 6
= C maneiras de escolher as comisses.

Pelo principio aditivo temos 120 + 15 = 135 maneiras de escolher as comisses.
10. O comprimento de uma palavra a quantidade de caracteres que ela possui. Encontre a
quantidade de palavras de comprimento 5 que podemos formar fazendo uso de 10 caracteres distintos,
de forma que no existam trs caracteres consecutivos idnticos em cada palavra.

Usando os 10 caracteres temos
5
10 maneiras de formar as palavras de comprimento 5.
So indesejveis as palavras cujo formato est mostrada na figura abaixo.

A A A B C

Grupo nico

O grupo que repete pode ser contado como 10 . 3! (so 10 caracteres diferentes e temos trs itens
para permutar AAA, B e C.
Para as duas posies que sobram podemos ter 9 . 8 = 72 maneiras de escolha (pois esses caracteres
no podem ser iguais).
Portanto temos
5
10 8 . 9 . ! 3 . 10 = 95680 palavras.


11. Quantos nmeros inteiros existem entre 1 e 10.000 que no so divisveis por 3; 5 e 7?

Obs.: Fiz o exerccio sem considerar que fossem mltiplos simultneos.

So 3333 mltiplos de 3, 1999 mltiplos de 5 e 1428 mltiplos de 7.

So 666 mltiplos de [3, 5] = 15; 476 mltiplos de [3, 7] = 21 e 285 mltiplos de [5, 7] = 35.

So 95 mltiplos de [3, 5, 7] = 105.

Pelo princpio da incluso e excluso temos 10000 [3333 + 1999 + 1428 (666 + 476 + 285) + 105] =
4562 nmeros que no so mltiplos de 3, 5 e 7.





12. Quantas so as permutaes da palavra PROPOR nas quais no existem letras consecutivas
iguais?

O mtodo usado foi atravs da fixao de uma das letras e determinando as possibilidades de
posicionar as demais letras sem haver repetio.
Cada linha da tabela abaixo mostra as posies escolhidas para uma letra.

Fixando o P dessa forma temos 2 . 1 . 1 . 1 = 2 possibilidades
Fixando o P dessa forma temos 2 . 1 . 2 . 1 = 4 possibilidades
Fixando o P dessa forma temos 2 . 1 . 1 . 1 = 2 possibilidades
Fixando o P dessa forma temos 2 . 1 . 1 . 1 = 2 possibilidades

Temos um total de 2 + 4 + 2 + 2 = 10 possibilidades usando a letra P.
Como so trs letras, ento temos 3 x 10 = 30 palavras nas quais no existem letras consecutivas
iguais.


13. De quantos modos 6 casais podem sentar-se ao redor de uma mesa circular de tal forma que
marido e mulher no fiquem juntos?

Primeiro posicionamos o grupo de mulheres que pode ser feito de 5! = 120 maneiras.
Em seguida devemos posicionar os homens observando a restrio imposta.
Designando os casais como (M
1
, H
1
); ... ; (M
6
, H
6
) sabemos que o H
i
no pode ficar ao lado de M
i
.
Na figura abaixo verificamos que existem duas posies proibidas para cada H
i
.



Ento podemos posicionar os homens de 4! = 24 maneiras distintas.
Logo, pelo principio multiplicativo, temos 24 x 120 = 2880 maneiras de posicionar os seis casais
obedecendo restrio imposta.
P 2 P 1 1 1
P 2 1 P 2 1
P 2 1 1 P 1
P 2 1 1 1 P
Proibido para H1
Proibido para H1
B R
4!
5! 5!
4! 3! 4!

5!
L
14. Quantas so as permutaes das letras da palavra BRASIL em que o B ocupa o primeiro lugar, ou
o R ocupa o segundo lugar, ou o L o sexto lugar?

No diagrama ao lado temos:

Anagramas em que o B ocupa o
primeiro lugar 5!.

Anagramas em que o R ocupa o
segundo lugar 5!.

Anagramas em que o L ocupa o sexto
lugar 5!.

Anagramas contados com repetio de B e R 4!.

Anagramas contados com repetio de B e L 4!.

Anagramas contados com repetio de L e R 4!.
Anagramas com as trs condies do problema 3!.
Portanto, pelo princpio da incluso e excluso, temos um total de 5! + 5! + 5! (4! + 4! + 4!) + 3! = 294
anagramas que contemplam o enunciado.

15. De quantas formas podemos representar o nmero 15 como soma de vrios nmeros naturais?

No encontramos um padro de combinatria para a soluo.
Foi resolvido, em sala, relacionando caso a caso e encontrou-se 176 formas.

16. Quantos quadrados perfeitos existem entre 40.000 e 640.000 que so mltiplos simultaneamente
de 3, 4 e 5?

Os mltiplos simultneos de 3, 4 e 5 so mltiplos de [3, 4 , 5] = 60 = 2
2
. 3 . 5. Seja M um nmero
natural mltiplo de 60 e que seja quadrado perfeito temos ento que M = 60.Q = Q . 5 . 3 . 2
2
, onde
N Qe de forma que tenhamos, no mnimo, Q =
2
7 . 5 . 3 e no mximo Q =
2 2
13 . 5 . 3 . 2 , pois pela
condio do problema M deve ser maior que 40000 e 60 .
2
7 . 5 . 3 =
2 2
7 . 5 . 3 . 5 . 3 . 2 =
2 2 2 2
7 . 5 . 3 . 2 = 44100
e 60 .
2 2
13 . 5 . 3 . 2 =
2 2 2
13 . 5 . 3 . 2 . 5 . 3 . 2 = 608400 que representam, respectivamente, o primeiro
quadrado perfeito mltiplo de 60 maior que 40000 e o maior quadrado perfeito mltiplo de 60 menor
que 640000.
Fazendo uma anlise dos nmeros quadrados perfeitos que cumprem o enunciado percebemos que
eles so da forma 900 .
2
P , como 900 . 7 900 . 49 44100
2
= = e 900 . 26 900 . 676 608400
2
= = ento
temos que 26 7 s s P . Portanto so 26 7 + 1 = 20 nmeros quadrados perfeitos que cumprem as
condies do enunciado.


17. Oito amigos vo ao cinema assistir a um filme que custa um real. Quatro deles possuem uma nota
de um real e quatro possuem uma nota de dois reais. Sabendo-se que o caixa do cinema no possui
nenhum dinheiro, como eles podem organizar uma fila para pagar o filme permitindo o troco pelo
caixa?



De acordo com a rvore de possibilidades acima temos 14 caminhos diferentes para o posicionamento
dos oito amigos.
O grupo que possui um real cada pode ser posicionado de 4! = 24 maneiras na fila, da mesma forma
os que possuem dois reais cada tambm podem ser posicionados de 4! = 24 maneiras na fila.
Ento, pelo principio multiplicativo, temos um total de 14 . 24. 24 = 8064 maneiras de dispor os amigos
na fila.

18. Se considerarmos todas as configuraes do tabuleiro com duas torres que no se atacam, como
no Exemplo 5.2, sem distinguir as torres, quantas configuraes obteremos?
No exemplo 5.2 as torres so diferentes e representam 3136 7 . 64
2
= maneiras distintas para
posicion-las de forma que no se ataquem. Para o caso proposto as torres so idnticas portanto
temos 1568
! 2
7 . 64
2
= configuraes.

19. Continuando o problema anterior, generalize-o para 3, 4, 5, ... torres que no se atacam,
encontrando tambm o nmero mximo de torres que podem ser colocadas no tabuleiro de modo que
duas delas no se ataquem.

Observando a soluo do problema 18 e a figura ao lado
temos que 64 representa o nmero de casas para colocar a
primeira torre, 7
2
representa o nmero de casas que a
segunda torre pode ocupar (posicionando a primeira sobra
uma quantidade de casas equivalente um quadrado com 7
casas de lado) e o ! 2 corrige o fato das torres serem iguais
(a permutao no gera uma nova disposio).
Para trs torres temos
! 3
6 . 7 . 8
2 2 2
configuraes, para
quatro torres temos
! 4
5 . 6 . 7 . 8
2 2 2 2
e para n torres temos
( ) | |
!
1 8 ... 7 . 8
2 2 2
n
n +


O mximo acontece para n = 8. Basta posicionar todas as torres na diagonal.


20. Tente fazer o problema anterior para cavalos de xadrez.

Iniciei com as figuras abaixo, mas no consegui concluir.






Cavalo nos vrtices.

Cavalo 1 4 opes
Cavalo 2 61 opes






Cavalo na segunda ou
stima coluna das laterais.

Cavalo 1 8 opes
Cavalo 2 60 opes






Cavalo na 3
a
ou 4
a
ou 5
a
ou
6
a
coluna das laterais.

Cavalo 1 16 opes
Cavalo 2 59 opes
21. Mostre que em toda sequncia de n
2
+1 inteiros distintos possui uma subsequncia crescente de n +
1 elementos ou uma subsequncia decrescente de n + 1 elementos.

Seja um inteiro positivo e suponhamos, por contradio, que haja uma sequncia de
2
+1 inteiros
distintos que no contenha uma subsequncia montona (cresceste ou decrescente) de tamanho +1,
ou seja, todas as subsequncias montonas de tem tamanho , no mximo. Seja um elemento da
sequncia . Associamos a um par ordenado de inteiros , , onde o comprimento da maior
subsequncia crescente de que comea em e o comprimento da maior subsequncia
decrescente de que comea em . Sobre as subsequncias montonas de podemos afirmar que:

) o mximo de pares ordenados de inteiros utilizados
2
. De fato, como o comprimento de uma
subsequncia montona de no mximo igual a , utilizamos inteiros de 1 a . Logo, utilizamos no
mximo .=
2
pares ordenados de inteiros;

) dois elementos distintos de no podem estar associados ao mesmo par ordenado de inteiros. De
fato, suponhamos que e so elementos de S, com . Seus pares ordenados so , e , e
como , temos que < ou >. Se <, afirmamos que >, pois sabemos que existe uma
subsequncia crescente de comprimento comeando em . Se inserirmos no comeo dessa
subsequncia, obtemos uma subsequncia crescente de comprimento +1. Assim, +1 ou,
equivalentemente, >. Assim e tm pares ordenados diferentes. Da mesma forma, se >, ento
> e, novamente e esto associados a pares ordenados diferentes.

Entretanto, essas duas afirmaes levam a uma contradio. H apenas
2
pares ordenados de inteiros
diferentes, e tem
2
+1 elementos. Pelo PCP (Principio da Casa dos Pombos), dois dos elementos
devem ter o mesmo par ordenado. Entretanto, isto contradiz o fato de que dois elementos quaisquer
no podem estar associados ao mesmo par ordenado. Portanto deve ter uma subsequncia
montona de comprimento +1.

22. Encontre o nmero de zeros que termina o nmero 2010!.

Usando a tcnica de contar os grupos das potncias de 5 (que multiplicados pelas potncias de 2
resulta em zeros) temos:

Temos 2010 : 5 = 402 grupos de 5
1
.

Temos 402 : 5 = 80 grupos de 5
2
.

Temos 80 : 5 = 16 grupos de 5
3
.

Temos 16 : 5 = 3 grupos de 5
4
.

Como 5
4
a maior potncia de 5 presente em 2010, temos um total de 402 + 80 + 16 + 3 = 503 zeros
na terminao de 2010!.


23. O jogo do 7 consiste em lanar dois dados e somar o nmero obtido nas suas faces. Caso a soma
seja 7, o jogador A ganha dois reais do jogador B. Caso a soma no seja 7, o jogador B ganha um real
de A. Pergunta-se: quem leva vantagem?

Quem leva vantagem o jogador B, pois no lanamento de dois dados temos 36 6
2
= somas
diferentes. Dentre estas apenas 6 do total sete. Portanto em cada jogada temos uma probabilidade de
6
1
a favor do jogador A e
6
5
a favor do jogador B.


24. A funo | de Euler associa a cada nmero natural n o valor |(n) igual ao nmero de inteiros
positivos menores ou iguais a n relativamente primos com n. Ou seja, ( ) { } 1 , ; 1 ) ( = s s = n m n m n |

Usando os princpios estudados, mostre que se n se decompe
em fatores primos como
k
k
p p p n
o o o
... .
2 1
2 1
= , ento
|
|
.
|

\
|

|
|
.
|

\
|

|
|
.
|

\
|
=
k
p p p
n n
1
1 ...
1
1 .
1
1 ) (
2 1
| .

|
|
.
|

\
|
|
|
.
|

\
|
|
|
.
|

\
|
=
|
|
.
|

\
|

|
|
.
|

\
|

|
|
.
|

\
|
=
k
k
k
p
p
p
p
p
p
n
p p p
n n
1
...
1
.
1
.
1
1 ...
1
1 .
1
1 ) (
2
2
1
1
2 1
|

( )( ) ( )
k
k
p p p
p p p
n n
... .
1 ... 1 . 1
. ) (
2 1
2 1

= | , substituindo
k
k
p p p n
o o o
... .
2 1
2 1
= temos:

( )( ) ( )
k
k
k
p p p
p p p
p p p n
k
... .
1 ... 1 . 1
. ... . ) (
2 1
2 1
2 1
2 1

=
o o o
| dividindo as potncias dos fatores primos de n resulta

( )( ) ( ) 1 ... 1 . 1 . ... . ) (
2 1
1 1
2
1
1
2 1
=

k k
p p p p p p n
k
o o o
|










Captulo 6 - Solues dos Exerccios

QUESTO 1
Se q
n
denota a soma q
n
= 1
2
+ 2
2
+ ... + n
2
, prove que para todo n N
6
) 1 2 1 + +
=
.n )( n(n
q
n

| |
6
) 3 2 )( 2 )( 1 (
) 1 ( ... 2 1
Logo,
6
) 3 2 )( 2 )( 1 (
6
)
2
3
)( 2 ( 2 ) 1 (
6
) 6 7 2 )( 1 (
6
) 1 ( 6 ) 1 2 ( ) 1 (
6
) 1 ( 6 2 2 1
6
) 1 ( 6 1 2 1
) 1 ( 2 1
) 1 (
6
1 2 1
) 1 ( 2 1
: teremos
6
2 2 1
2 1 de membros dois nos ) 1 ( Somando
: o Demonstra
6
) 3 2 )( 2 )( 1 (
) 1 ( ... 2 1
: ento , 1 para induo, de Tese
6
) 2 2 )( 1 (
... 2 1
: ento , com para a verdadeir que induo, de Hiptese - 2
1
6
) 1 1 2 1 1 1
1 1
: pois , 1 para a verdadeir afirmao A - 1
finita. induo do provar Vamos
2 2 2 2
2 2
2
2 2 2 2
2 2 2 2 2
2 2 2 2
2 2 2 2
2 2 2
2
+ + +
= + + + + +
+ + +
=
+ + +
=
+ + +
=
+ + + +
=
+ + + +
+ + + +
= + + + + +
+ +
+ +
= + + + + +
+ +
= + + + +
+ + +
= + + + + +
+ =
+ +
= + + +
e =
=
+ +
= = =
=
k k k
k k
k k k
k k k
k k k k k k k k ) k )( k(k
k ) k )( k(k
k k ...
k
) k )( k(k
k k ...
) k )( k(k
k ... k
k k k
k k
k n
k k k
k
N k k n
. )( (
q q
n
n n


QUESTO 2
Use o princpio da induo para provar as seguintes afirmaes:
(a) 3
2n+1
+ 2
n+2
divisvel por 7 para todo n N;
7. por divisvel Que
) 2 3 . 3 ( 7 3 . 6 14 3 . 27 )
4
3 . 3 7
( 8 3 . 27 2 3
: teremos hiptese a do substituin , 2 . 8 3 . 27 2 3
teremos Tese, a ndo Desenvolve
: o Demonstra
7 por divisvel 2 3
: ento , 1 para induo, de Tese
4
3 7
2 logo , 7 2 3 ento 7, por divisvel 2 3
: ento , com para a verdadeir que induo, de Hiptese - 2
7. por divisvel que 35 2 3 2 3
: pois , 1 para a verdadeir afirmao A - 1
finita. induo do provar Vamos
2 2 2
2
2 3 3 2
2 3 3 2
3 3 2
1 2
2 1 2 2 1 2
3 3 2 1 1 1 . 2
q q
q
k n
q
q
N k k n
n
k k k
k
k k k
k k k k
k k
k
k k k k k
+ = + =

+ = +
+ = +
+
+ =

= = + +
e =
= + = +
=
+ +
+ +
+ +
+
+ + + +
+ +


(b) a soma dos cubos de trs nmeros naturais consecutivos divisvel por 9;
Demonstrar que n
2
+ (n+1)
2
+ (n+2)
2
divisvel por 9.
9. por divisvel Que
) 9 9 3 ( 9 ) 3 ( ) 2 ( ) 1 (
27 27 9 9 ) 3 ( ) 2 ( ) 1 (
) 3 ( ) 1 ( 9 ) 1 ( ) 3 ( ) 2 ( ) 1 (
: que concluimos na tese, ) 2 ( do substituin
) 1 ( 9 ) 2 (
teremos, hiptese na ) 2 ( Isolando
: o Demonstra
9 por divisvel ) 3 ( ) 2 ( ) 1 (
: ento , 1 para induo, de Tese
9 ) 2 ( ) 1 ( ento 9, por divisvel ) 2 ( ) 1 (
: ento , com para a verdadeir que induo, de Hiptese - 2
9. por divisvel 36 3 2 1
: pois , 1 para a verdadeir afirmao A - 1
finita. induo do provar Vamos
2 3 3 3
2 3 3 3 3 3
3 3 3 3 3 3 3
3
3 3 3
3
3 3 3
3 3 3 3 3 3
3 3 3
+ + + = + + + + +
+ + + + = + + + + +
+ + + + + = + + + + +
+
+ = +
+
+ + + + +
+ =
= + + + + + + + +
e =
= + +
=
n n q n n n
n n n n q n n n
n n n q n n n n
n
n n q n
n
n n n
k n
q n n n n n n
N k k n
n


1 1
Vamos provar do induo finita.
1 - A afirmao verdadeira para 1, pois:
(10 9.1 10) 81
7 7. 7. 7
81 81
2 - Hiptese de induo, que verdadeira para com , ento:
7 77 777 ... 777...7
k vezes
n
n k k N
+

=

= = =
= e
+ + + +
1
2 2
1
1
(10 9 10)
7.
81
Tese de induo, para 1, ento:
(10 9( 1) 10) (10 9 19)
7 77 777 ... 777...7 7. 7.
81 81
Demonstrao:
Somando 777...7 nos dois membros da hiptese tere
k
k k
k vezes
k vezes
k
n k
k k
+
+ +
+
+

=
= +
+
+ + + + = =
1
1 1
1 1
1 1
1
mos
(10 9 10)
7 77 777 ... 777...7 777...7 7. 777...7
81
(10 9 10) (10 9 10)
7. 777...7 7. 7.111...1
81 81
7.111...1 7.(10 ...
k
k vezes k vezes k vezes
k k
k vezes k vezes
k
k vezes
k
k k
+
+ +
+ +
+ +
+

+ + + + + = +

+ = +
= +
n 1
2 2
1 1 1
2
1 1
2
1
x 1
10 10 1), lembrando que ... 1, ento
1
10 1 10 1 9 9.10 9
7.(10 ... 10 10 1) 7. 7. . 7.
10 1 10 1 9 81
(10 9 10) (10 9 10)
7. 7.111...1 7. 7.(10 ... 10 10 1)
81 81

n
k k k
k
k k
k
k vezes
x x x
x
k k
+
+ + +
+ +
+

+ + + = + + + +


+ + + + = = =


+ = + + + + +
1 1 1 2
(10 9 10) 9.10 9 (10.10 9 10 9) (10 9 19)
7. 7. 7. 7.
81 81 81 81
k k k k
k k k
+ + + +

+ = =




1
Vamos provar do induo finita.
1 - A afirmao verdadeira para 1, pois:
( 1)( 2)( 3)...( ) 2 .1.3.5...(2 1)
(1 1) 2 2 .1.(2.1 1) 2.1 2
2 - Hiptese de induo, que consideraremos verdadeira par
n
n
n n n n n n
=
+ + + + =
+ = = = =
1
1
a com , ento:
( 1)( 2)( 3)...( ) 2 .1.3.5...(2 1)
Tese de induo, para 1, ento:
( 1 1)( 1 2)( 1 3)...( 1 1) 2 .1.3.5...2( 1)
( 2)( 3)( 4)...( )(2 1)(2 2) 2 .1.3.5...(
k
k
k
n k k N
k k k k k k
n k
k k k k k k
k k k k k k k
+
+
= e
+ + + + =
= +
+ + + + + + + + + = +
+ + + + + + =
hiptese de induo
2 1)
Demonstrao:
Vamos trabalhar com o primeiro mebro da tese
( 2)( 3)( 4)...( )(2 1)(2 2)
( 2)( 3)( 4)...( )(2 1).2( 1) ( 1)( 2)( 3)( 4)...( ).2(
k
k k k k k k k
k k k k k k k k k k k k k
+
+ + + + + + =
+ + + + + + = + + + + +
1
2 1)
Substituindo a hiptese teremos :
2 .1.3.5...(2 1).2(2 1) 2 .1.3.5...(2 1).(2 1) . Temos a validade de nossa tese.
k k
k
k k k k
+
+ =
+ = +


QUESTO 3
Use o princpio da induo para provar as seguintes desigualdades:

A desigualdade ( ) ( )
n n n n
b a b a + > +
1
2 no vlida para todo n natural, pois ela no vlida para
1 = n . Para 1 = n , ela fica b a b a + > + , que no verdadeiro. Mas, de fato, a desigualdade
( ) ( )
n n n n
b a b a + > +
1
2 vlida para todo 2 > n . Ela vlida para 2 = n , pois para 2 = n , ela fica
( ) ( )
2 2 2
2 b a b a + > + , que equivale a
2 2 2 2 2
2 2 2 ab b a b a + + > + , que equivale a 0 2
2 2
> + ab b a ,
que equivale a ( ) 0
2
> b a , que verdadeiro, j que b a = . Suponha que a igualdade vale para n e
vamos mostrar que ela vale para 1 + n , isto , vamos mostrar que ( ) ( )
1 1 1
2
+ + +
+ > +
n n n n
b a b a , isto ,
vamos mostrar que ( ) ( )
1 1 1
2
+ + +
+ < +
n n n n
b a b a . De fato, como ( ) ( ) ( ) b a b a b a
n n
+ + = +
+1
, 0 > +b a e,
por hiptese de induo, ( ) ( )
n n n n
b a b a + < +
1
2 , ento ( ) ( )( ) b a b a b a
n n n n
+ + < +
+ 1 1
2 . Assim, para
concluir que ( ) ( )
1 1 1
2
+ + +
+ < +
n n n n
b a b a suficiente mostrar que ( )( ) ( )
1 1 1
2 2
+ +
+ < + +
n n n n n n
b a b a b a .
Mas, ( )( ) ( )
1 1 1
2 2
+ +
+ < + +
n n n n n n
b a b a b a equivale a ( ) ( )( ) b a b a b a
n n n n
+ + > +
+ + 1 1
2 , que equivale a
n n n n n n
ab b a b a b a + + + > +
+ + + + 1 1 1 1
2 2 , que equivale a
n n n n
ab b a b a + > +
+ + 1 1
, que equivale a
n n n n
ab b a b b a a + > + , que equivale a ( )( ) 0 > b a b a
n n
, que verdadeiro, pois se b a > , ento
0 >
n n
b a e 0 > b a , sendo portanto ( )( ) 0 > b a b a
n n
, e se b a < , ento 0 <
n n
b a e 0 < b a ,
sendo portanto ( )( ) 0 > b a b a
n n
.






A desigualdade n
n
> + + +
1
2
1
1
1
no vlida para todo n natural, pois ela no vlida para
1 = n . Para 1 = n , ela fica 1
1
1
> , que no verdadeiro. Mas, de fato, a desigualdade
n
n
> + + +
1
2
1
1
1
vlida para todo 2 > n . Ela vlida para 2 = n , pois para 2 = n , ela fica
2
2
1
1
1
> + , que equivale a 2
2
1 2
>
+
, que equivale a 2 1 2 > + , que verdadeiro. Suponha
que a igualdade vale para n e vamos mostrar que ela vale para 1 + n , isto , vamos mostrar que
1
1
1 1
2
1
1
1
+ >
+
+ + + + n
n n
. De fato, como, por hiptese de induo,
n
n
> + + +
1
2
1
1
1
, ento
1
1
1
1 1
2
1
1
1
+
+ >
+
+ + + +
n
n
n n
. Assim, para concluir que
1
1
1 1
2
1
1
1
+ >
+
+ + + + n
n n
suficiente mostrar que 1
1
1
+ >
+
+ n
n
n . Mas,
1
1
1
+ >
+
+ n
n
n equivale a
( )
1
1
1 1
+ >
+
+ +
n
n
n n
, que equivale a ( ) 1 1 1 + > + + n n n , que
equivale a ( ) n n n > +1 , que equivale a ( )
2
1 n n n > + , que equivale a 0 > n , o que verdadeiro, pois
2 > n .


A desigualdade
24
13
2
1
2
1
1
1
> + +
+
+
+ n n n
no vlida para todo n natural, pois ela no vlida para
1 = n . Para 1 = n , ela fica
23
13
2
1
> , que no verdadeiro. Mas, de fato, a desigualdade
24
13
2
1
2
1
1
1
> + +
+
+
+ n n n
vlida para todo 2 > n . Ela vlida para 2 = n , pois para 2 = n , ela fica
24
13
4
1
3
1
> + , que equivale a
24
13
12
7
> , que verdadeiro. Suponha que a igualdade vale para n e vamos
mostrar que ela vale para 1 + n , isto , vamos mostrar que
24
13
2 2
1
1 2
1
2
1
3
1
2
1
>
+
+
+
+ + +
+
+
+ n n n n n
. De fato, como, por hiptese de induo,
24
13
2
1
2
1
1
1
> + +
+
+
+ n n n
, ento
1
1
24
13
2
1
2
1
+
> + +
+ n n n
. Como
1
1
24
13
2
1
2
1
+
> + +
+ n n n
,
ento
2 2
1
1 2
1
1
1
24
13
2 2
1
1 2
1
2
1
3
1
2
1
+
+
+
+
+
>
+
+
+
+ + +
+
+
+ n n n n n n n n
. Assim, para concluir que
24
13
2 2
1
1 2
1
2
1
3
1
2
1
>
+
+
+
+ + +
+
+
+ n n n n n
suficiente mostrar que
24
13
2 2
1
1 2
1
1
1
24
13
>
+
+
+
+
+

n n n
. Mas,
24
13
2 2
1
1 2
1
1
1
24
13
>
+
+
+
+
+

n n n
equivale a
2 2
1
1 2
1
1
1
+
+
+
<
+ n n n
, que equivale a
( )( ) 2 2 1 2
3 4
1
1
+ +
+
<
+ n n
n
n
, que equivale a
( )( ) ( )( ) 3 4 1 2 2 1 2 + + < + + n n n n , que equivale a 3 7 4 2 6 4
2 2
+ + < + + n n n n , que equivale a 0 1> + n , o
que verdadeiro.



QUESTO 4

Mostre a seguinte identidade trigonomtrica
2
( 1) cos cos( 1) 1
cos 2cos 2 .... cos
2
4
n nx n n x
x x n nx
x
sen
+ +
+ + + =
Vamos considerar a identidade
2
2(1 cos )
2
4
x
x
sen
= , logo nossa identidade a ser demonstrada pode
ser reescrita com o
P(n):
( 1) cos cos( 1) 1
cos 2cos 2 .... cos
2(1 cos )
n nx n n x
x x n nx
x
+ +
+ + + =


Vamos demonstrar P(n) utilizando induo finita.
Vamos inicialmente testar n = 1, ento
2
2cos 2 s(2) 1
(1) cos cos (V)
2(1 cos ) 2(1 cos ) 2(1 cos )
2cos (1 cos )
2cos 2cos
x co x
P x x
x x x
x x
x x

= = = = =


Hiptese: Vamos supor que P(n) verdadeira para n = k
( 1) cos cos( 1) 1
( ) cos 2cos 2 .... cos
2(1 cos )
k kx k k x
P k x x k kx
x
+ +
= + + + =


Tese: Vamos demonstrar que P(n) verdadeira para n=k+1.
( 2) cos( 1) ( 1) cos( 2) 1
( 1) cos 2cos 2 .... cos ( 1) cos( 1)
2(1 cos )
k k x k k x
P k x x k kx k k x
x
+ + + +
+ = + + + + + + =


Somando (k+1)cos(k+1)x aos dois membros da hiptese teremos:
| |
( 1) cos cos( 1) 1
cos 2cos 2 .... cos ( 1) cos( 1) ( 1) cos( 1)
2(1 cos )
( 1) cos cos( 1) 1 2( 1) cos( 1) 2cos ( 1) cos( 1)
2(1 cos )
cos( 1) 2 2 2( 1) cos ( 1) cos 1
2(1
k kx k k x
x x k kx k k x k k x
x
k kx k k x k k x x k k x
x
k x k k k x k kx
+ +
+ + + = + + = + + + =

+ + + + + + +
=

+ + + + + +

| |
| |
( )
cos( 1) 2 2( 1) cos ( 1) cos 1
cos ) 2(1 cos )
( 2) cos( 1) ( 1) 2cos cos( 1) cos 1
2(1 cos )
( 2) cos( 1) ( 1) cos( 2) ( 1) (2cos cos( 1) cos cos( 2) ) 1
2(1 cos
I
k x k k x k kx
x x
k k x k x k x kx
x
k k x k k x k x k x kx k x
+ + + + +
= =

+ + + +
=

+ + + + + + +
) x
=



Vamos provar que (I) = 0.
Primeiramente observe que:
cos(k+2)x=cos[(k+1)x+x]=cos(k+1)x.cosx-sen(k+1)x.senx (II)
e
coskx=cos[(k+1)x-x]=cos(k+1)x.cosx+sen(k+1)x.senx (III)
Ento trabalhando com (I) teremos que
2cos cos( 1) cos cos( 2)
2cos cos( 1) (cos(k+1)x.cosx+sen(k+1)x.senx+cos(k+1)x.cosx-sen(k+1)x.senx)=
2cos cos( 1) 2cos cos( 1) 0
Com isso provamos que a identidade trigonomtrica verdad
x k x kx k x
x k x
x k x x k x
+ + =
+
+ + =
eira.


QUESTO 5

Um torneio de xadrez tem n jogadores. Cada jogador joga uma nica partida com cada um dos outros
jogadores. Calcule o nmero total de partidas realizadas no torneio.

O nmero de jogos realizados no torneiro ser dado pela
( )
2
n
n !
2 2! 2 !
C
n
n
| |
= =
|

\ .
. Observe que so
n jogadores, a partida realizada entre dois jogadores e cada dupla joga uma nica vez.
Vamos considerar P(n) como sendo o total de jogos realizados, ou seja que
P(n) =
( )
!
2! 2 !
n
n
. Vamos demonstrar a validade de P(n) para todo n natural maior que 1, utilizando
induo finita.

Para n = 2 fcil verificar que ser uma nica partida.


( )
2!
(2) 1
2! 2 2 !
P = =



Como hiptese vamos considerar que P(n) vlida para n = k, com k natural e maior que 2.


( )
! k.(k-1)
( )
2! 2 ! 2
k
P k
k
= =



Tese: Vamos demonstrar que para n = k+1, P(n) tambm verdadeira.


( )
( 1)! (k+1).k
( 1)
2! 1 ! 2
k
P k
k
+
+ = =


Para demonstrar a validade de P(k+1) devemos perceber inicialmente que se para k jogadores o
nmero de jogos dado por
.( 1)
( )
2
k k
P k

= (hiptese de induo), ao aumentarmos um jogador ,
teremos k jogos a mais pois, esse novo jogador ir jogar com os k outros jogadores.
Somando k hiptese teremos:
2 2
( )
.( 1) ( 1)
2
2 2 2 2
P k
k k k k
k k k k k
k
+
= + = = =
+
+

Temos ento a validade de nossa tese.






QUESTO 6

| |
2
3 3 3 3
2
2 2 2 2 3 2 2
3
2
3
2
3 3 3 3 3
2
2
3 3 3 3
2
3 3 3 3
2
2
3
2
) 2 )( 1 (
) 1 ( ... 3 2 1
Logo.
2
) 2 )( 1 (
4
) 2 ( ) 1 (
4
4 4 ) 1 (
4
) 1 ( 4 ) 1 (
) 1 (
2
) 1 (
) 1 (
2
) 1 (
) 1 ( ... 3 2 1
: temos hiptese da membros dois nos 1 Somando
: o Demonstra
2
) 2 )( 1 (
) 1 ( ... 3 2 1
: ento , 1 para induo, de Tese
2
) 1 (
... 3 2 1
: ento , com para a verdadeir que induo, de Hiptese - 2
1 1
2
) 1 1 .( 1
1
: pois , 1 para a verdadeir afirmao A - 1
finita. induo do provar Vamos
(

+ +
= + + + + +
(

+ +

+ +
=
+ + +
=
+ + +
= + +
(

+
+ +
(

+
= + + + + + +
+
(

+ +
= + + + + +
+ =
(

+
= + + + +
e =
= =
(

+
=
=
k k
k
k k
k k k k k k k k
k
k k
k
k k
k k
) (k
k k
k
k n
k k
k
N k k n
n


QUESTO 7


3
2
1
3
2
1
1
2
1
1 1
) 1 (
1
1
1
1
). 1 ( 1
) 1 (
1
1
1
1
1
: temos Newton de binmio do mento desenvolvi Pelo
: o Demonstra
3
1
1
1
: ento , 1 para induo, de Tese
3
1
1 3
1
1
: ento , com para a verdadeir que induo, de Hiptese - 2
3 2
1
1
1
: pois , 1 para a verdadeir afirmao A - 1
finita. induo do provar Vamos
1
1 1
2
1
2
1
0
1
1
1
1
1
1
< = + = + + s
s
+
|
|
.
|

\
| +
+
+
+ + =
+
|
|
.
|

\
| +
=
|
.
|

\
|
+
+ =
s
|
.
|

\
|
+
+ =
+ =
= +
|
.
|

\
|
+ s
|
.
|

\
|
+ =
e =
s =
|
.
|

\
|
+ =
=
+
+ +
=
+
=
+
=
+
+
+
+


k
k
n
k
k
n
k
n
k
n
n
k
n
n
n
n
n
k k
k
k n
k
k
k
k n
k
n
a
n
a
k n
q
k k
a
N k k n
a
n


QUESTO 8

2
1 4 1
...
: logo ,
2
1 4 1
2
1 4 2 1
Sendo,
2
1 4 2 1
...
2
1 4 1
...
2
1 4 1
...
: hiptese da membros dois aos a somar vamos ento 0, a Sendo
: o Demonstra
2
1 4 1
...
: ento , 1 para induo, de Tese
2
1 4 1
...
: ento , com para a verdadeir que induo, de Hiptese - 2
2
1 4 1
: pois , 1 para a verdadeir afirmao A - 1
finita. induo do provar Vamos
1
1
1
1
+ +
< + + + + +
+ +
<
+ + +
+ + +
< + + + + +
+ +
+ < + + + + +
+ +
+ < + + + + +
<
+ +
< + + + +
+ =
+ +
< + + + +
e =
+ +
<
=
+
+
+

a
a a a a a
a a a
a a
a a a a a
a
a a a a a a
a
a a a a a a
a
a a a a
k n
a
a a a a
N k k n
a
a
n
radicais k
radicais k
radicais k
radicais k
radicais k
radicais k
radicais k























QUESTO 9

133. por divisvel Que
) 11 1728 ( 133 ) 11 . 11 1728 ( 133 12 11
11 . 11 . 144 1728 . 133 11 . 11
12
11 133
. 1728 11 . 1331 12 11
: teremos hiptese a do substituin , 12 . 1728 11 . 1331 12 11
teremos Tese, a ndo Desenvolve
: o Demonstra
133 por divisvel 12 11
: ento , 1 para induo, de Tese
12
11 133
2 1 logo , 133 12 11 ento 133, por divisvel 12 11
: ento , com para a verdadeir que induo, de Hiptese - 2
133. por divisvel que 133 . 23 12 11 12 11
: pois , 1 para a verdadeir afirmao A - 1
finita. induo do provar Vamos
2 2 3 2 3
2 3
2
3 2 3
2 3 2 3
3 2 3
2
2 1 2 2 1 2 2
3 3 1 1 . 2 2 1
+ + +
+
+ +
+ +
+ +
+
+ + + +
+ +
= = +
+ =
|
|
.
|

\
|
+ = +
+ = +
+
+ =

= = + +
e =
= + = +
=
k k k k
k k
k
k k k
k k k k
k k
k
k k k k k
q q
q
q
k n
q
q
N k k n
n


QUESTO 10


7. por divisvel Que
) 2 3 . 3 ( 7 3 . 6 14 3 . 27 )
4
3 . 3 7
( 8 3 . 27 2 3
: teremos hiptese a do substituin , 2 . 8 3 . 27 2 3
teremos Tese, a ndo Desenvolve
: o Demonstra
7 por divisvel 2 3
: ento , 1 para induo, de Tese
4
3 7
2 logo , 7 2 3 ento 7, por divisvel 2 3
: ento , com para a verdadeir que induo, de Hiptese - 2
7. por divisvel que 35 2 3 2 3
: pois , 1 para a verdadeir afirmao A - 1
finita. induo do provar Vamos
2 2 2
2
2 3 3 2
2 3 3 2
3 3 2
1 2
2 1 2 2 1 2
3 3 2 1 1 1 . 2
q q
q
k n
q
q
Z k k n
n
k k k
k
k k k
k k k k
k k
k
k k k k k
+ = + =

+ = +
+ = +
+
+ =

= = + +
e =
= + = +
=
+ +
+ +
+ +
+
+ + + +
+
+ +





Questo 11
2n+2 + 6 1
Mostre que para todo n temos que um mltiplo de 11.
3 Z 2
n+
e +
Vamos fazer a demonstrao por induo finita.
Consideremos n = 0.
Substituindo teremos
2.0+2 2 6.0 1 1
= + = que mltiplo de 11.
3 3 2 2 11
+
+
Hiptese de Induo - Suponhamos que seja verdade para n=k , com k
+

Z
e , temos que
2.k+2 6. 1 +
= com q
11q
3 2 Z
k +
+ e
Tese
Vamos demonstrar que para n = k+1 ,
2.k+4 6. 7

11 divide 3 2
k +
+
Desenvolvendo nossa tese teremos
2.k+4 2k+2 2 6. 7 6 7
= . .
3 3 3 2 2 2
k k +
+ + .
Agora iremos trabalhar com a hiptese.
2.k+2 2.k+2 6. 1 6. 1
= ( )
11q 11q
3 3 2 2
k k
I
+ +
+ =
Substituindo (I) na tese teremos que
2 6 6 6 7 6 1 6 7 6 6
6 1
( ). . . . )
9.11 9. 9.11 9.11 11(9 10. 11 3 18.2 128 110.2 2 2 2 2 2 2 2
2
k k k k k k k
k
q q q q q
+
+
+ = + = + = + =
+

Como
6
) divisvel por 11, provamos que a tese tambm verdadeira.
11(9 10.
2
k
q+

Logo provamos que
2n+2 6 1 +
11 divide para todo n .
3 2 Z
n+
+ e


Questo 12
Considere
n F
a sequncia de Fibonacci. Mostre que
n
. .
1 1
5 5
1 1
F
2 2
5 5
n n
=
| | | |
+
| |
| |
\ . \ .

Um soluo:

A sequncia de Fibonacci corresponde recorrncia
1 2 1 2
, com 1
n n n F F F F F
= + = = . Temos
ento que essa sequncia 1,1,2,3,5,8,13,21,34,.......
Vamos demonstrar a validade de
n
. .
1 1
5 5 1 1
F
2 2 5 5
n n
=
| | | |
+
| |
| |
\ . \ .
, que define cada termo dessa
sequncia. Vamos utilizar induo finita.
Para n = 1
Temos que
1 1
1
. . .
1 5 1 5 1 5 1 5
1 1 1
1
2 2 2
5 5 5
F
| |
|
= = =
|
|
\ .
| | | |
+ + +
| |
| |
\ . \ .

Como hiptese, consideremos que
n F
vlido para n = k.
n = k temos que
. .
1 5 1 5
1 1
2 2
5 5
k k
k F
=
| | | |
+
| |
| |
\ . \ .

Nossa tese ser provar que para n = k+1 ,
n F
tambm vlida.
Para n = k+1 ,
1 1
1
. .
. . . . .
1 5 1 5
1 1
2 2
5 5
1 5 1 5 1 5 1 5
1 1
2 2 2 2
5 5
k k
k
k k
F
+ +
+
= =

| | | |
+
| |
| |
\ . \ .
| | | | | | | |
+ +
| | | |
| | | |
\ . \ . \ . \ .

Temos que
1 1 k k k F F F +
= + ,
Somando
1 1
1
1 1
. . . .
. . . . . .
. .
1 5 1 5 1 5 1 5
1 1 1 1
2 2 2 2
5 5 5 5
1 5 1 5 1 5 1 5 1 5 1 5
1 1 1 1
2 2 2 2 2 2
5 5 5 5
1 5 1
1 1
2
5 5
k k k k
k k
k k k k
k k
F F


+ = + =
+ =

| | | | | | | |
+ +
| | | |
| | | |
\ . \ . \ . \ .
| | | | | | | | | | | |
+ + +
| | | | | |
| | | | | |
\ . \ . \ . \ . \ . \ .
| |
+
|
|
\ .
. . . .
. . . .
. . . .
2 2 5 1 5 1 5
1 1
2 2 2 1 5 1 5
5 5
1 5 1 5
1 2 1 2
1 1
2 2
5 1 5 5 1 5
1 5 1 5 3 5 3 5
1 1
2 2
5 1 5 5 1 5
k k
k k
k k
+ =
( (
( (
+ =
( (
( (

| | | |
|

|
|
\ . \ .
| | | | | |
| | | |
+
| | |
| |
| | |
+
\ . \ .
\ . \ . \ .
| | | |
+
| | +
| |
+
\ . \ .
| | | |
+ +
| |
| |
+
\ . \ .
1
. . . .
1 5 1 5 1 5 1 5
1 1
2 2 2 2
5 5
k k
k F +
|
=
|
|
| | | |
| |
=
| |
| |
\ . \ .
| | | |
+ +
| |
| |
\ . \ .

Outra soluo
Consideremos as progresses geomtricas
n
n
q
v
= , com 0 q = , que satisfaz recorrncia
1 2 1 2
, com 1
n n n x x x x x
= + = =
.
Temos que
1 2

cujas solues so
1 5 1 5
e
2 2
q q
n n n
q q q

= +
+
= =

Defina

n
e
w
n m
n
q q
v
= = .
Como
n
e
w n v
satisfazem recorrncia
1 2 1 2
, com 1
n n n x x x x x
= + = =
,

n

1 2
ento, para todo e reais, a sequncia tambm satisfaz recorrncia.
Agora impomos 1, o que nos d um sistema de duas equaes com as duas
1
incgnitas e , cujas solues so =
5
F
F
n
n
w
v
F
o |
o | o
|
o
= +
= =
1
e =- .
5
|


Questo 13
Mostre as seguintes propriedades a respeito da sequncia de Fibonacci
n F
:
a)
2
1
1
n
i n
i
F F +
=
=

b)
2 1 2
1
n
i n
i
F F
=
=



c)
2 2 1
1
1
n
i n
i
F F +
=
=

d)
2
1 1
.
( 1)
n
n n n F F F +
=


Resoluo.
a)
2
1
1
n
i n
i
F F +
=
=


Para n = 1 temos que
1 3 1 1 F F
= =
Hiptese: Vamos supor que seja verdade para n = k com k natural e maior que 1
1 2 2
............
1 k k F F F F +
+ + + =
Tese: Vamos demonstrar que a frmula verdadeira para n = k+1
1 2 1 3
k+2 1 3 3 1 k+2
............
Substituindo a hiptese no primeiro membro da igualdade temos que:
, o que verdade pois,
1
F 1 1 F
k k k
k k k k
F F F F F
F F F F
+ +
+ + + +
+ + + + =
+ = = +

b)
2 1 2
1
n
i n
i
F F
=
=


Para n = 1 temos:
1 2 1 F F
= =

Verdadeiro
Como hiptese vamos supor a validade da frmula para n = k, sendo k natural maior que 1
1 3 2 1 2
.............
k k F F F F
+ + + =


Tese: Vamos demonstrar que a frmula verdadeira para n = k+1.
Para n = k+1 temos
1 3 2 1 2 1 2 2
.............
k k k F F F F F + +
+ + + + =

Considerando a tese temos que
1 3 2 1 2 1 2 2
.............
k k k
hiptese
F F F F F + +
+ + + + =

Substituindo a hiptese, ento
2 2 1 2 2 k k k F F F + +
+ =

O que mostra a validade da Tese.


c)
2 2 1
1
1
n
i n
i
F F +
=
=


Vamos demonstar a validade desta propriedade utilizando induo finita.
Para n = 1 vlida a propriedade.
2 3 1 2 1 1 F F
= = =

Como hiptese de induo vamos considerar que essa propriedade vlida para n = k , com k
natural e maior que 1.
Para n = k teremos:
2 4 2 2 1
......... 1
k k F F F F +
+ + + =

Tese : Vamos demonstrar que a propriedade vlida para n = k+1
Para n = k+1
2 4 2 2 2 2 3
......... 1
k k k F F F F F + +
+ + + + =

Trabalhando com a tese

2 4 2 2 2 2 3
hiptese de induo
......... 1
k k k F F F F F + +
+ + + + =

Substituindo a hiptese de induo
2 1 2 2 2 3 2 1 2 2 2 3
1
1 k k k k k k F F F F F F + + + + + +
+ = + =

O que demonstra a validade de nossa tese.

d)
2
1 1
.
( 1)
n
n n n F F F +
=


Para n= 2 temos a validade da propriedade.
2
2
1 3 2
2
. .
( 1)
1.2 1
1
F F F
=

= =


Hiptese: Vamos supor que a propriedade vlida para n = k com k natural e maior que 2.
Para n = k temos
2
1 1
.
( 1)
k
k k k F F F +
=



Tese: Vamos demonstrar que a propriedade tambm vlida para n = k+1.
Para n = k+1 temos que:
1
2
2 1
.
( 1)
k
k k k F F F
+
+ +
=



Para demonstrar a validade da tese, vamos inicialmente trabalhar com a hiptese:

2
1 1
.
( 1)
k
k k k F F F +
=


Multiplicando os dois membros por (-1) obtemos
( )
1
2
1 1
2
1
1 1
1
2 2
1 1 1 1 1 1
1
2 2
1 1 1 1
.
.
1 1
. .
.
( 1)
( 1)
( 1)
2
( 1)
3
k
k k k
k
k k
k
k k k k k k
k
k k k k
k k
F F F
F F F F
F F F F F F
F F F F
+
+
+
+
+
+ + +
+
+ +
+ =
+ =
+
+ + =
+ =



Agora iremos trabalhar com o primeiro membro da tese
( ) ( )
2 2
2 1 1 1 1 1 1
2 2 2
1 1 1 1 1 1 1
2 2 2 2 2 2 2
1 1 1 1 1 1 1 1 1 1 1
hiptese de induo
. .
.
2
2 2
3 3 2
k k k k k k k k
k k k k k k k
k k k k k k k k k k k
F F F F F F F F
F F F F F F F
F F F F F F F F F F F
+ + + + +
+ + + +
+ + + + + + +
= =
+ =
+ = +
+

Substituindo a hiptese de induo na tese, obtemos:
1 1
2 2
1 1
( 1) ( 1)
k k
k k F F
+ +
+ +
+ =


Com isso demonstramos a validade da nossa tese.

Questo 14
De quantas formas diferentes podemos cobrir um tabuleiro de 2 x n com peas de domins que cobrem
exatamente duas celas do tabuleiro?

Considere um tabuleiro com 2 linhas e n+2 colunas. Para preencher o canto esquerdo do tabuleiro, h
duas alternativas: colocar um domin em p , restando um tabuleiro com 2 linhas e n+1 colunas a
preencher( iremos preencher o que falta de x
n+1
maneiras) ou colocar dois domins deitados restando
um tabuleiro com 2 linhas e n colunas (que poder ser preenchido de x
n
maneiras distintas). Logo, o
nmero x
n
de modos de preencher um tabuleiro 2 x n com domins 2 x 1 satisfaz a recorrncia
x
n+2
= x
n+
x
n+1
, com x
1
= 1 e x
2
= 2. Esta a sequncia de Fibonacci, logo temos

1 1
1 1 5 1 1 5
2 2
5 5
n n
n n x F
+ +
= =
| | | |
+
| |
\ . \ .





Questo 15
. Calcular o nmero de regies em que o plano dividido por n retas distintas em cada uma das
seguintes situaes:

(a) as n retas so concorrentes;
(b) no existem duas retas paralelas nem trs retas concorrentes

a) As n retas concorrentes dividem o plano em 2n regies.
Vamos demonstrar tal fato por induo finita.

Para n = 1, ou seja uma reta, o plano fica dividido em 2.1 = 2 regies.

Vamos supor que seja verdade que para n = k o plano fica dividido em 2k regies ( Hiptese de
induo)

Vamos verificar que para n = k+1 o plano fica dividido em 2(k+1) regies.
Temos que em um conjunto de k retas, cada reta acrescentada (deve ser uma reta concorrente
s outras retas) aumenta mais duas regies. Ou seja para k retas temos 2k regies,
acrescentando mais uma reta teremos 2k + 2 regies e 2k+2=2(k+1) que demonstra nossa tese.

c)Fazendo alguns desenhos que satisfazem ao problema verificamos que:




O nmero de regies P(n) em que o plano fica dividido por n retas, sendo duas no paralelas e
nem trs concorrentes dado por
n(n+1)
P(n)= 1
2
+ .

Vamos demonstrar a validade de P(n) utilizando induo finita.


Inicialmente vamos considerar n = 1. Vemos facilmente que para uma reta o plano fica dividido
em duas regies.

1.(1+1)
P(1)= 1 2
2
+ =

Hiptese: Vamos considerar que para n = k seja validade P(k) =
k.(k+1)
P(k)= 1
2
+ .
Nmero de retas (n)
Nmero de regies
P(n)
1 2
2 4
3 7
4 11
5 16
Tese: Vamos demonstrar que para n = k+1 seja vlida P(k+1).
Considerando que para k retas o plano fica dividido em
(k+1).(k+2)
P(k)= 1
2
+ . Ao acrescentarmos mais
uma reta ela acrescenta k+1 novas regies. Observe que a nova reta ao encontrar a primeira reta, ela
separa em duas regies a regio em que est, entrando em outra regio. Ao encontrar a segunda reta,
ela separa em duas regies a regio em que est, entrando em outra reta, e assim sucessivamente at
encontrar a n-sima reta. Assim so obtidas k+1 regies a mais das que j existiam, logo:
P(k+1) = P(k) + k+1 =
2
2
k.(k+1)
1 k+1=
2
( 1).( 2) 3 2
1 1 1 1
2 2 2
k k k
k k k
k
+
+ + + +
+ + + +
= + + + =
. C.q.d
Questo 16
Dizemos que uma figura enquadrvel com rgua e compasso, se a partir dela possvel, utilizando
apenas rgua e compasso, construir um quadrado de mesma rea. Prove que:
(a) um tringulo sempre enquadrvel;
(b) um polgono qualquer enquadrvel.

Sugesto para o item (b): Utilize induo dividindo a figura em tringulos.

a+b+c
um tringulo de lados a,b e c. Temos que A= p(p-a).(p-b).(p-c) sendo p= .
2
Seja x = p.(p-a).(p-b).(p-c)
Seja


















Podemos construir um quadrado com lado L = h que ter rea igual a A= p(p-a).(p-b).(p-c) .


b) Considere P(n): Todo polgono de n lados enquadrvel.

Base de induo.

P(3) verdadeira pelo item anterior.

Suponhamos que P(k) vlida.

Supondo P(k) verdadeira, vamos provar que isto implica em P(k+1) verdadeira.

Todo polgono com (k+1) lados, ao traarmos uma diagonal conveniente, particionado em um
tringulo e um polgono com k lados. Sabemos construir um quadrado Q
1
que tem rea igual a de um
tringulo e sabemos construir Q
2
, um quadrado com rea igual a rea de um polgono de k lados (pela
X 1
Temos que h
2
= x . 1

h
h 1
Podemos escrever L
2
= h . 1

L
hiptese de induo). Seja f o lado de Q
1
e g o lado do quadrado Q
2.
Temos que f
2
a rea do quadrado
Q
1
e g
2
a rea do quadrado Q
2
. Pelo teorema de Pitgoras temos que f
2
+ g
2
= m
2
, onde m
2
a rea
de um novo quadrado com lado m.


Questo 17

D uma resposta situao Observao 6.17.

Sugesto: Observe a validade do argumento quando o conjunto A tem 2 elementos. Veja que B e C no
se intersectam. Ou seja, o passo indutivo falha de n = 1 para n = 2.

O erro se encontra no passo indutivo de n=1 para n=2.

Para n=1 verdade, pois se h uma camisa todas tem a mesma cor.
Agora vamos considerar n = 2, ou seja h duas camisas. No posso afirmar que as duas tem a mesma
cor, o que foi considerado como verdadeiro na demonstrao na observao 6.17.
Vamos tentar demonstrar que se n = 1 verdadeiro ento n =2 tambm verdadeiro.

Para n = 2 .
Consideremos o conjunto B = {b
1,
b
2
}
Vamos dividir em dois subconjuntos
B

= { b
1
} e B ={ b
2
} , temos que B = B

U

B
fato que no podemos garantir que
B

B C


































Captulo 7 Solues dos Exerccios


1 Provar que em todo tringulo a soma dos comprimentos das medianas menor que o permetro do
tringulo e maior que o semipermetro deste.
Observe a figura, onde o tringulo ABC tem lados a, b e c e as medianas tem medidas m
a,

(mediana relativa ao lado a ) ,m
b
(mediana relativa ao lado b) e m
c
(mediana relativa ao lado c)

Vamos provar inicialmente que a soma dos comprimentos das medianas de um tringulo maior que o
semipermetro deste, ou seja:


Somando as desigualdades obtemos:
c.q.d



Agora iremos provar que em um tringulo a soma dos comprimentos das medianas menor que
o permetro do tringulo. Prolonguemos, por exemplo a mediana mb e a partir do vrtice A traamos
um segmento de comprimento a e paralelo ao lado BC que intersectar o prolongamento de mb em D.
Faremos processos anlogos s medianas mc e ma



Temos que:
2
2
2
b
a
c
a c
b c
a b
m
m
m
< +
< +
< +
Somando as desigualdades teremos:
a b c
a b c
m m m
+ + < + + c.q.d

2
2
2
2
a b c
c
b
c
a b c
Temos
c
b
b
a
a
b
m m m
m
m
m
+ +
+ + >
+ >
+ >
+ >
2
a b c
a b c
m m m
+ +
+ + >

2 Os centros de trs crculos que no se intersectam esto sobre uma reta. Prove que se um quarto
crculo toca de forma tangente os trs crculos, ento o raio deste maior que pelo menos um dos raios
dos trs crculos dados.

O caso em que uma das trs circunferncias tangente internamente circunferncia de raio R j
satisfaz a condio de R ser maior que o raio de pelo menos uma das circunferncias.

Vamos analisar agora outra situao.

Considere a figura em que as trs circunferncias com centros sobre uma reta t, so exteriores duas a
duas e tangentes externas quarta circunferncia. Seja C1 de raio R1, C2 de raio R2 e C3 de raio R3.
A quarta circunferncia tem raio R e centro C. Considere x o espaamento entre as circunferncias 3 e
2 e y, o espaamento entre as circunferncias 2 e 1


Pela figura, considerando o tringulo de lados R + R
3
, R + R
1
e R
1
+x+2R
2
+y+R
3
, podemos escrever:


1 2 3 1 3
2
2
2
,
2 2
2 2
2 2
y
x R R R R R R
y
x R R
R R
R R
+ + + + < + +
+ + <
<
>


CQD.


3 Dado n inteiro positivo, provar que
1
1 2
1
n
j
n
j n
=
>
+


Vamos utilizar o fato que a mdia aritmtica de um conjunto de nmeros reais maior que ou igual a
mdia harmnica desses valores.
1 1 1 1
.......
2
1 2 ....
MA MH
n
n
n
n
>
+ + +
>
+ + +
, multiplicando a desigualdade por n, escrevendo 1+2+ ....+ n como
n(n+1)/2, obtemos:
1 1 1 2
.......
2 1
n
n n
+ + + >
+
que podemos escrever como
1
1 2
1
n
j
n
j n
=
>
+

c.q.d


4 A soma de trs nmeros positivos 6. Provar que a soma deseus quadrados no menor que 12.

Sejam a, b e c nmeros reais positivos.
Temos que a + b + c = 6.
Vamos utilizar o fato de que
MA MQ s
2 2 2
2 2 2
2 2 2
2 2 2
3 3
2
3
4
3
12
a b c
a b c
a b c
a b c
a b c
+ +
+ +
s
+ +
s
+ +
s
s + +


5 Determinar as dimenses do paraleleppedo de menor diagonal possvel, sabendo que a soma dos
comprimentos de todas suas arestas 12.

Considere um paraleleppedo retngulo de dimenses a, b e c, como o comprimento de todas suas
arestas 12, podemos escrever, 4a + 4b + 4c = 12, ento a+ b+ c = 3.
Seja d a diagonal, sabendo que d =
2 2 2
a b c
+ + e usando o fato de que MAMQ, teremos que:
2 2 2
2 2 2
2 2 2
2 2 2
3 3
1
3
1
3
3
a b c
a b c
a b c
a b c
a b c
+ +
+ +
s
+ +
s
+ +
s
s + +

Como queremos a menor diagonal, tomaremos d = 3 e sabemos que a igualdade MA = MQ, ocorre
quando a=b=c e neste caso teremos a = b= c = 1.


6 Encontrar todas as solues positivas do sistema de equaes no lineares

Considerando a primeira equao do sistema podemos escrever que a mdia aritmtica desses valores
:
2 2 2
1 2 10
..............
1
10 10
MA
x x x
+ +
= =
+
e utilizando a segunda equao do sistema, podemos escrever
que
2 2 2
1 2 10
10 10 1
1 1 1
100 10
..........
MH
x x x
= = =
+ + +
.

Temos que:
2 2 2
1 2 10
...... MA MH
x x x
= = = =
Substituindo na primeira equao, obtermos
2
1 1
10 1 1/ 10
x x
= = , mas queremos solues
positivas, logo
1 2 10
1
......
10
x x x
= = = =

7 Demonstrar que, se
1 2
, ,.....,
n a a a
so nmeros positivos tais que

1 2
. ...... 1
n a a a
=
Ento (1+a
1
) . (1+a
2
) ......... (1+a
n
) > 2
n.

Suponhamos por absurdo que (1+a
1
) . (1+a
2
) . (1+a
n
) < 2
n
. Dividindo por 2
n
, obtemos:
1 2
(1 ) (1 ) (1 )
. ........... 1
2 2 2
n a a a
+ + +
< .
Iremos usar o fato de que MGMA. Observe que para todos os a
i
s, com i {1, 2,..... } n e , teremos que
1
1.
2
i
i i
a
a a
+
> = , logo:
1 2
. ........... 1
n
n a a a
< ,
Sabemos que
1 2
. ...........
n a a a
= 1 (hiptese do problema),
Teramos o fato de 1< 1 , o que um absurdo. Logo vale a desigualdade
(1+a
1
) . (1+a
2
) ......... (1+a
n
) > 2
n.

8 Prove que a mdia geomtrica super-aditiva, isto , para nmeros no negativos
i i
e , 1 i n, tem-se
a b
s s

1 1 1
( )
n n n
n n n
i i i
i
i i i
a
a b b
= = =
+ s +
[ [ [


Alm disso, estude em que condies ocorre a igualdade.
Sugesto: Use a desigualdade entre as mdias geomtrica e aritmtica.

Temos que MAMG, logo
1 1
1 1
1 1 1 1
1
1 1
1
1
(I)
(II)
Fazendo (I) ( )

n
)
(
n
n
i i
n
i i
i i i i
n
n
i i
n
i i
i i i i
n n
n n
i i i i
n n
i i i i
i i i i i i i i
n
n n
i i
n n
i i
i
i i i i
n
n
i
i
i
i
i
n
n
II
n
a a
a b a b
b b
a b a b
a b a b
a b a b a b a b
a b
a b
a b
a
b
a
n
n
= =
= =
= = = =
=
= =
=
=
+ +
>
+ +
>
+
+ +
+ + + +
>
+
+
+
>
+
>
[
[
[ [

[ [
[
1 1 1
1 1 1
1 1
1 ) c.q.d
) )
(
( (
n n n n
n n n n
i i i n n n
i i i
n n n
i i i
i n n
i i i
n n
i i
i i
i i
b a b
a
b a b
a a
b b
= = =
= = =
= =
+ +
> + > +
+ +
[ [ [ [
[ [ [
[ [



9 - Usar o mtodo de induo para provar a desigualdade de Cauchy-Schwarz.

Considere P(n) =
2 2 2 2 2 2
1 1 2 2 1 2 1 2
........... ........ . ........
n n n n ab a b a b a a a b b b
+ + + s + + + + + +

Vamos demonstrar que P(n) vlida para todo n natural, e para tal utilizaremos a induo finita.
Para n = 1 verificamos que P(n) vlida. Observe :
2
2 2
1 1 1 1 1 1
( )
a b a b a b
s =

Hiptese de induo: Suponhamos que P(n) seja verdadeira para n = k, ento teremos
2 2 2 2 2 2
1 1 2 2 1 2 1 2
........... ........ . ........
P(k)=
k k k k ab a b a b a a a b b b
+ + + s + + + + + +

Tese:
Vamos provar que para n=k+1 , P(n) verdadeira.
Sendo n = k+1, temos que
1 1 2 2 1 1 1 1 2 2 1 1
2 2 2 2 2 2 2 2 2 2 2 2 2 2
1 2 1 2 1 1 1 2 1 1 2 1
........... ...........
........... . ........... . ..... . ..... .
k k k k k k k k
k k k k k k
a b a b a b a b a b a b a b a b
a a a b b b a b a a a b b b
+ + + +
+ + + +
+ + + + s + + + + s
+ + + + + + + s + + + + + +

Vamos verificar a validade desta ltima desigualdade.
Considere x, y, z e w nmeros reais positivos. Utilizando o fato de que MGMA podemos escrever:
2
2 2
. 2 . ( 2 .
2
2 . ( ).( ) ( ).( )
. . . .
( ) ( ) ( )
xw yz
xw yz xw yz xw yz xy zw xw yz xy xw yz zw
xw yz y w x z y w x z
x y z w x y y w
xy zw xy zw
+
s s + + + s + + +
+ + s + + + s + +
+ s + +


2 2 2 2 2 2 2 2
1 2 1 2 k+1 k+1
Basta considerarmos ........... , y = ........... , z = e w=
a b x k k a a a b b b
+ + + + + +
=

e a desigualdade fica demonstrada.


10 Para todo real
2
1
0
( )
n
i
i i
a b

=
>
+

Use este fato para dar outraprova da desigualdade de Cauchy-Schwarz

Podemos escrever:

2 2
2 2 2 2 2
1 1 1 1 1
2. . . ) 2. . 0
( ) (
n n n n n
i i i i i i i
i
i i i i i
i i
a
ab b a a b b a b


= = = = =
= + + = + + >
+

Fazendo:

2
1
n
i
i
v
b
=
=

, u =
1
.
n
i i
i
a b
=

e t =
2
1
n
i
i
a
=

,
Obtemos a desigualdade do segundo grau em .
2
0
2 t u v
+ >
+

Temos que v > 0 e 0 As
2
2
2
4 4 0
4 4 0
4 4
u vt
u vt
u vt
A =

s
s
s

Ento teremos que
2
.
.
u vt
u vt
s
s

Que a desigualdade de Cauchy-Schwarz, com
2
1
n
i
i
v
b
=
=

, u =
1
.
n
i i
i
a b
=

e t =
2
1
n
i
i
a
=

,

11 Use a desigualdade de Cauchy-Schwarz para dar uma prova alternativa da desigualdade entre as
mdias aritmtica e quadrtica (ma mq).

A desigualdade de Cauchy-Schwarz nos diz que
2 2
1 1 1
.
n n n
i i i i
i i i
ab a b
= = =
s

, fazendo 1,
i b
= a
desigualdade poder se reescrita como
2
1 1
.
n n
i i
i i
n
a a
= =
s

que equivale a
2
2 2
1 2 2
1
.......... ......... ).
(
n n
n
a
a a a a a
+ + + s + + + ,
Dividindo por n, os dois membros da desigualdade j que n um nmero natural e positivo, podemos
escrever:
2 2 2
1 2 1 2
......... ........
n n
n n
a a a a a a
+ + + + + +
s , logo MA s MQ . cqd

12 Prove que
1 1 1
1
2 2
2
n n n
i i
i i i
i i
ab
a b
= = =
(
s +
(



Uma soluo

Queremos provar que
1 1 1
1
2 2
2
n n n
i i
i i i
i i
ab
a b
= = =
(
s +
(



Temos que
1 1
1 1
2 2
1
2 2
2 2
n n
n n
i i
i i
i i
i i
a b
a b
= =
= =
+
(
+ =
(



, que consideraremos a mdia aritmtica de dois
termos.
A desigualdade de Cauchy-Schwarz nos diz que
2 2
1 1 1
.
n n n
i i i i
i i i
ab a b
= = =
s

,
Usando o fato de que MG MA s
2 2
1 1
.
1 1 1 1 1
2 2
1
2 2
.
2 2
n n
n n n n n
i i
i i i i
i i i i i
i i
i i
a b
a b a b
a b
= =
= = = = =
+
(
s s + =
(







13 Prove que a
4
+ b
4
+ c
4
abc(a + b + c)

Uma soluo , considerando a, b e c reais.

Inicialmente vamos considerar o fato de que MQMA
Podemos escrever
2 2 2
2
2 2 2
2
2 2 2
3.( )
3 3
( )
( )
3 9
a b c a b c
a b c
a b c
a b c
a b c
+ +
>
+ +
> + + >
+ +
+ +
+ +
,
Desenvolvendo esta ltima desigualdade concluiremos que a
2
+ b
2
+ c
2
ab+ ac + bc .
A partir desta concluso podemos escrever a
4
+b
4
+c
4
=(a
2
)
2
+ (b
2
)
2
+(c
2
)
2
a
2
b
2
+ a
2
c
2
+ b
2
c
2
(I)
Temos o fato de que a
2
b
2
+ a
2
c
2
+ b
2
c
2
(ab+ac+bc)
2
/3. (II)
Para justificarmos a desigualdade II basta observarmos que
2 2 2
2 2 2 2 2 2
2 2
2 2
( ) ( ) 2.( ).
2 2 2
3 3 3
ab ac ab ac bc ab ac bc
bc ab a b a c abc b c b c
a c
+ + + +
= =
+ + + + +
+ +
=
2 2 2 2 2 2 2 2 2
.
2
3 3
a b a b b c a bc ab c abc
+ + + +
+
2 2 2 2 2 2
a b a b b c
+ + .
Vamos trabalhar com a desigualdade II, temos que
a
2
b
2
+ a
2
c
2
+ b
2
c
2
(ab+ac+bc)
2
/3 que equivale a
2 2 2 2 2 2
a b a c b c
+ +
2 2 2 2 2 2 2 2 2
.
2
3 3
a b a b b c a bc ab c abc
+ + + +
+
2 2 2
( )
( )
III
abc a b c
a bc ab c abc
+ + =
+ +
.
Pela desigualdade I temos que

4 4 4 2 2 2 2 2 2
a b c a b a c b c
+ + > + +

e a desigualdade III nos fornece que
2 2 2 2 2 2
( ) abc a b c
a b a c b c
+ + >
+ +

Logo, conclumos que
a
4
+ b
4
+ c
4
abc(a + b + c)


Outra soluo considerando a, b e c 0

Inicialmente vamos considerar o fato de que MAMG
4 4
4 4 4 4 2 2
4 4
4 4 4 4 2 2
4 4
4 4 4 4 2 2
. .
. .
. .
2
2
2
2
2
2
a b
a b a b a b
a c
a c a c a c
b c
b c b c b c
+
> + >
+
> + >
+
> + >

Somando estas desigualdades
4 4 4 2 2 2 2 2 2
4 4 4 2 2 2 2 2 2
2 2 2 2 2 2
( )
a b c a b a c b c
I
a b c a b a c b c
+ + > + +
+ + > + +

Utilizando novamente a desigualdade entre as mdias aritmtica e geomtrica podemos escrever:
2 2 2 2
2 2 2 2 2 2 2 2 2
2 2 2 2
2 2 2 2 2 2 2 2 2
2 2 2 2
2 2 2 2 2 2 2 2 2
. .
. .
. .
2
2
2
2
2
2
a b b c
a b b c a b b c a b c
a b a c
a b a c a b a c a b c
a c b c
a c b c a c b c a b c
+
> + >
+
> + >
+
> + >





Somando as desigualdades obtemos
2 2 2 2 2 2 2 2 2
2 2 2 2 2 2 2 2 2
2 2 2 2 2 2
( )
2 2 2 2 2 2
( )
II
a b a c b c a bc ab c abc
a b a c b c a bc ab c abc
abc a b c
a b a c b c
+ + > + +
+ + > + +
+ + >
+ +


A desigualdade I nos diz que
4 4 4 2 2 2 2 2 2
a b c a b a c b c
+ + > + +

e a desigualdade II nos fornece o fato de que
2 2 2 2 2 2
( ) abc a b c
a b a c b c
+ + >
+ +

Comparando I e II conclumos que
4 4 4
( ) abc a b c
a b c
+ + >
+ +
c.q.d



14 Prova que se a0, b0 e c 0. Ento (a + b)(a + c)(b + c) 8abc:

Vamos usar o fato de que MA MG >

. 2.
2
a c
a c a c ac
+
> + >
2
b c
bc b c bc
+
> + >
Multiplicando as inequaes, obtemos:
(a+b) . (a+c) . (b+c) 8.a.b.c c.q.d


15 Queremos provar que (1+x)
n
> 1+nx para todo x positivo e n natural ,

Uma soluo
Temos que (1+x)
n
> 1+nx (1 ) 1
n
x nx + > +

Sendo n natural e maior que um.
Considere os nmeros positivos a
1
, a
2, ......
a
n,
tais que

a
1.
a
2. ...... .
a
n
= 1 + nx (produto dos nmeros)
e
a
1+
a
2+ .....+.
a
n =
n + nx
Podemos dizer que um dos a
i
, com i
e
{1,2,...n} igual a 1+nx. Vamos supor que a
1
= 1 + nx, logo
teremos que a
2
=a
3
=........=a
n
= 1. Usando o fato de que MGMA,

1 1 1 1
(1 )
n
n n
nx n
nx nx x nx
n
x
+
+ s + s + + s
+
, como a ocorre para a
1
=a
2
=.....=a
n

E nossos valores no sero todos iguais a um teremos o que queramos demonstrar

(1+x)
n
> 1+nx


Outra soluo
Agora iremos demonstrar tal desigualdade utilizando induo.

(1+x)
n
> 1+nx
Observe que para n = 1 teremos (1+x)
1
= 1 + 1.x, ou seja uma igualdade, logo no vale para n = 1.
Iremos fazer a nossa induo para todo n natural e maior que 1.

Ento faremos o teste para n = 2
(1+x)
2
> 1+2x , vamos inicialmente desenvolver (1+x)
2
= 1 + 2x + x
2
, como x positivo, ento x
2
> 0,
logo
1+2x + x
2
> 1 + 2x, o que prova a desigualdade para n = 2.

Agora como hiptese, consideremos que para um n=k, com k natural e maior que 1, a desigualdade
(1+x)
k
> 1+kx seja verdadeira .

Vamos verificar se a hiptese verdadeira, implica que para n = k + 1 a desigualdade se torna verdadeira
(tese).
A nossa tese ser ento para n = k+1
(1+x)
k+1
> 1+(k+1)x.

A partir da hiptese, podemos multiplicar os dois membros da desigualdade por (1+x), pois se x
positivo, ento 1+x tambm ser positivo.
(1+x)
k
> 1+kx (1+x)
k+1
> (1+kx) (1+x). Agora vamos trabalhar com (1+kx).(1+x) = 1+x + kx + kx
2
=
1+(k+1)x + kx
2
,
como kx
2
positivo, temos que 1+(k+1)x + kx
2
> 1+(k+1)x, logo (1+x)
k+1
> 1+(k+1)x. c.q.d




16- Prove que se a; b; c e d so inteiros positivos, ento:

Vamos utilizar o fato de que MHMA.
Sejam quatro nmeros inteiros positivos a, b, c e d. Temos que:
4
a b c d
MA
+ + +
= e
4
1 1 1 1
MH
a b c d
=
+ + +
, logo

4 1 1 1 1
( ).( ) 16
1 1 1 1
4
a b c d
a b c d
a b c d
a b c d
+ + +
s + + + + + + >
+ + +



17 - Prova que se a0, b0 e c 0, ento

Vamos usar o fato de que MA MG >
Consideremos a, b e c trs nmeros maiores que ou iguais a zero.
Temos que
2
2
ab bc
b ac
acb
+
> =
2
2
ba ac
a bc
bca
+
> =
2
2
ac cb
c ab
abc
+
> =
Somando as desigualdades, obtemos
( )
2
2
ab bc ac
a bc b ac c ab
+ +
= + +
( ) ab bc ac a bc b ac c ab + + = + + c.q.d







18 - Prove que se x 0, ento 3x
3
- 6x
2
+ 4 0.
Sugesto: Use a desigualdade entre as mdias aritmtica e geomtrica.

Da inequao 3x
3
- 6x
2
+ 4 0, podemos escrever 3x
3
+ 4 6x
2

Vamos utilizar o fato de que MAMG.
Considere os monmios 2x
3
, x
3
e 4
Temos que


3
4
3
3
MA
x
+
= e
6 2
3
8 2
MG
x x
= =
Ento,
3
2
3 2 3 2
4
3
3
2
3 6 3 6 0 4 4
x
x
x x x x
+
>
+ + >
>



19 - Prove que se x 0, ento
8
2 4
3
x x + >
Vamos utilizar o fato de que MQMG.
Considere os monmios 3x e 4, temos que
2
2
2
(3 )
4
12
MQ
e
MG
x
x
+
=
=
ento podemos escrever
2
2
2
2 2
(3 )
4
9 16
12 12 6
x
x
x x x
+
+
> > >

Teremos ento que
2
2
2
2
2
2
.(4)
: (3)
. .
9 16 12
36 64 48
36 48 64 0
36 96 144 64 0
36 96 64 144
(6 8)
144
6 8 12
8
2 4
3
c q d
x x
x x
x x
x x x
x x x
x
x
x x
x x
+
+
+
+ +
+ +
>

>
>
>
>
>
+
+ >
+ >











20 Sejam C
1
e C
2
dois crculos concntricos de raios r
1
e r
2
, respectivamente, com r
1
< r
2
. Sobre o
crculo C
1
se marcam dois pontos P
1
e P
2
diametralmente opostos. Deseja-se encontrar o ponto P sobre
o crculo C
2
que maximiza a soma d(P) = PP
1
+ PP
2
.



Pela desigualdade das mdias temos que MA < MQ.

( ) ( )
2 2
2
2
2
1 2 1
PP PP PP PP +
s
+


( ) ( )
2
. 2 ) (
2
2
2
1
2 1
PP PP
PP PP P d
+
s + =

( ) ( )
2
2
2
1 2 1
. 2 ) ( PP PP PP PP P d + s + =

2 1
) ( PP PP P d
MAX
= , ou seja, o tringulo
2 1
PP P issceles.
Pela concluso anterior temos que o tringulo POP
1
retngulo, portanto:

( ) ( ) ( )
2
1
2
1
2
PP OP OP = + ento ( )
2
1
2
2
2
1
PP r r = +


2 2
1 2
( ) 2.
MAX
d P r r = +










Captulo 8 Solues dos Exerccios
QUESTO 1
Calcule o quociente e o resto da diviso de p(x) por q(x) para os polinmios p(x) e q(x) dados:
(a) p(x) = 3x
3
- 2x + 1 e q(x) = -7x - 1;
3x
3
+ 0x
2
- 2x + 1 -7x-1
- 3x
3
-3/7x
2
-3/7x
2
- 3/49x + 101/343 quociente

- 3/7x
2
2x + 1
+3/7x
2
- 3/49x
-101/49x+1
+101/49x -101/343
Resto -> 242/343


(b) p(x) = x
5
- 1 e q(x) = x - 1;
x
5
+ 0x
4
+ 0x
3
+ 0x
2
+ 0x - 1 x - 1
- x
5
+ x
4
x
4
+ x
3
+ x
2
+ x quociente
x
4
+ 0x
3

- x
4
+ x
3

x
3
+0x
2

- x
3
+ x
2

x
2
+ 0x
- x
2
+ x
x - 1
-x + 1
Resto 0

(c) p(x) = 3x
5
- 2x
3
+ 1 e q(x) = x
2
+ x + 1
3x
5
+ 0x
4
- 2x
3
+ 0x
2
+ 0x + 1 x
2
+ x + 1
- 3x
5
- 3x
4
3x
3
3x
3
- 3x
2
- 2x + 5 quociente
- 3x
4
- 5x
3
+ 0x
2

+ 3x
4
+ 3x
3
+ 3x
2

- 2x
3
+ 3x
2
+ 0x
+ 2x
3
+ 2x
2
+ 2x
5x
2
+ 2x + 1
- 5x
2
- 5x - 5
Resto - 2x - 4


QUESTO 2
Encontre os valores de A e B de forma que

Reduzindo a expresso ao mesmo denominador e igualando os coeficientes do 1 e 2 membros,
temos:
2 1 1
1
1
1
1
1 ) ( ) 1 ( 1
1
1
2 2 2 2 2
= = +

=
= +

=
= +

+
=

+
=

+ =

+
B B
A
B A
A
B A
x x
x
x x
Bx A Ax
x x
x B x A
x x
x
x
B
x
A
x x
x

Ento A=-1 e B=2
QUESTO 3
Se os polinmios x
2
-x+4 e (x-a)
2
+(x+b) so iguais, encontre a + b.
Sendo os polinmios idnticos ento iremos igualar os coeficientes do 1 e 2 membros de acordo com
os seus expoentes

( )
( )

= = + = +
= = = +
+ + + + = +
+ + + = + + + = +
3 4 1 4
1 2 2 1 1 2
1 2 4
2 4 4
2
2 2 2
2 2 2 2 2
b b b a
a a a
b a x a x x x
b x a ax x x x b x a x x x


Ento a=1 e b=3

QUESTO 4
Quais os valores de a e b que tornam iguais os polinmios
P
1
(x) = x
2
- x - 6 e P
2
(x) = (x + a)
2
- b?
Sendo os polinmios idnticos ento iremos igualar os coeficientes do 1 e 2 membros de acordo com
os seus expoentes:

( )

= = =
|
.
|

\
|
=
= =
+ + = + = =
4
25
4
1
6 6
2
1
6
2
1
1 2
2 6 6
2
2
2 2 2 2 2
2 1
b b b b a
a a
b a ax x x x b a x x x P P



QUESTO 5
A diviso de P(x) por x
4
+ 1 tem quociente x + 2 e resto 1. Encontre o polinmio P(x).
Usando o principio fundamental da diviso teremos:

3 2 ) (
1 2 2 ) (
1 ) 2 )( 1 ( ) (
) ( ) ( ) ( ) (
4 5
4 5
4
+ + + =
+ + + + =
+ + + =
+ =
x x x x P
x x x x P
x x x P
x R x Q x D x P


QUESTO 6
Qual o resto da diviso do polinmio x
100
por x + 1?
De acordo com o teorema do resto temos que R = P(raiz do divisor), ento:

1
) 1 (
) 1 (
100
=
=
=
R
R
P R




QUESTO 7
Determine o resto da diviso do polinmio p(x) pelo polinmio
g(x) = x, onde p(x) = (x - 1)(x - 2) ... (x - n) + b .
Usando o teorema do resto teremos:
b n R
b n R
b n R
P R
n
+ =
+ =
+ =
=
! . ) 1 (
) )...( 3 )( 2 )( 1 (
) 0 )...( 3 0 )( 2 0 )( 1 0 (
) 0 (


QUESTO 8
Mostre que x
n
- 1 divisvel por x - 1 para todo 1 > n
Teremos que R= (1)
n
1 , como (1)
n
= 1 para n maior ou igual a 1, ento R = 1 1, R= 0 logo x
n
- 1
divisvel por x 1.

QUESTO 9
Faa os seguintes itens:
(a) encontre o quociente da diviso de x
n+1
- 1 por x - 1
1 ... ) ( Ento
. que temos Euclides de algoritmo Pelo
) ( ) 1 )( ... ( 1 que Sabemos
1
1 1
+ + + + =
+ + + + =

+
x x x x q
(x) ).d(x) + r P(x) = q(x
x r x x bx ax x
n n
n n n

(b) utilize a diviso anterior para calcular a soma 1 + x + x
2
+x
3
+ ... + x
n
dos n primeiros termos de uma
progresso geomtrica de razo x.
Usando a soma de termos de uma progresso geomtrica teremos:
| |
1
1
1
1 ) ( . 1
... 1
1 1
1 2


= + + + + +
+ +

x
x
x
x
x x x x
n n
n n

QUESTO 10
Determine o valor de a para que o polinmio P(x) seja divisvel
por x - a, onde P(x) = x
3
+ (1 - a)x
2
+ (1 + a)x - 1
1 2
1
) (
2
2 3 2 3
+ =
+ + + =
=
a a R
a a a a a R
a P R


Sendo P(x) divisvel por x-a ento R = 0.

0
1
0 1 2
0
2
1
2
=
=
= +
=
a
a
a a
R


QUESTO 11
Mostre que o polinmio P(x) = x
100
- 2x
50
+ 1 divisvel por x
2
- 1.

Se P(x) for divisvel por x
2
1, ento ele divisvel por x + 1 e x 1, pois
x
2
1 = (x + 1)(x 1).
Ento teremos: P(1) = 0 e P(-1) = 0

= +
= +
=

= +
= +
=
verdade
P
verdade
P
0 1 2 1
0 1 ) 1 ( 2 ) 1 (
0 ) 1 (
0 1 2 1
0 1 ) 1 ( 2 ) 1 (
0 ) 1 (
50 100 50 100



QUESTO 12
Mostre que o resto r(x) da diviso do polinmio p(x) por x - s r(x) = p(s).

Na diviso do polinmio p(x) por x s, teremos quociente de grau n-1, para p(x) de grau n e o resto ter
grau 0, logo o resto uma constante que chamaremos de r.
Usando a equao fundamental da diviso teremos:

r x q s x x p + = ) ( ). ( ) (


Tomando x = s, ento:
) (
) ( . 0 ) (
) ( ). ( ) (
s p r
r s q s p
r s q s s s p
=
+ =
+ =


Logo r = p(s) que chamado de Teorema de DAlambert.

QUESTO 13
Dado o polinmio p(x) = a
n
x
n
+ a
n-1
x
n-1
+ ... + a
1
x + a
0
definimos a derivada de p(x) como sendo o
polinmio, p'(x) = na
n
x
n-1
+ (n - 1)a
n-1
x
n-2
+ ... + 2a
2
x + a
1
:
Por exemplo, a derivada do polinmio x
5
o polinmio 5x
4
e a derivada do polinmio x
3
+5x
2
+2x-1 o
polinmio 3x
2
+10x+2.
Usando as informaes, calcule:

(a) a derivada dos polinmios:
(i) x + 1;

1 ) ( ' = x p


(ii) x
4
+ 3;


3
4 ) ( ' x x p =

(ii) 1 + x + x
2
+ x
3
+ ... + x
n
.
1 2
... 3 2 1 ) ( '

+ + + + =
n
nx x x x p

(b) Sabendo que p(0) = 1, calcule tambm o polinmio p(x) cuja derivada
(i) x
4
.

1
5
1
) (
5
+ = x x p


(ii) -x
2
+ 1.

1
3
1
) (
3
+ + = x x x p

(ii) x
3
+ 2x
2
+ 3.

1 3
3
2
4
1
) (
3 4
+ + + = x x x x p

(c) Prove que se p(x) e q(x) so polinmios, ento
(i) (p + q)(x) = p(x) + q(x)
Dado
0 1
2
2
1
1
... ) ( a x a x a x a x a x p
n
n
n
n
+ + + + + =

e
0 1
2
2
1
1
... ) ( b x b x b x b x b x q
n
n
n
n
+ + + + + =


Derivando p(x) e q(x) teremos
x b x b x b n x nb x q
x a x a x a n x na x p
n
n
n
n
n
n
n
n
1 2
2
1
1
1 2
2
1
1
2 ... ) 1 ( ) ( '
2 ... ) 1 ( ) ( '
+ + + + =
+ + + + =


Somando as derivadas:
x b a x b a x b a n x b a n x q x p
n
n n
n
n n
) ( ) ( 2 ... ) )( 1 ( ) ( ) ( ' ) ( '
1 1 2 2
2
1 1
1
+ + + + + + + + = +


Somando os polinmios e derivando a soma dos polinmios temos:
) ( ) ( 2 ... ) )( 1 ( ) ( ) ( )' (
) ( ) ( ) ( ... ) ( ) ( ) )( (
1 1
1
2 2
2
1 1
1
0 0 1 1
2
2 2
1
1 1
b a x b a x b a n x b a n x q p
b a x b a x b a x b a x b a x q p
n
n n
n
n n
n
n n
n
n n
+ + + + + + + + = +
+ + + + + + + + + + = +



Logo teremos que:
) ( ' ) ( ' ) ( )' ( x q x p x q p + = +

(ii) (pq)(x) = p(x)q(x) + p(x)q(x)

QUESTO 14

QUESTO 15
(a) 1 + x
2
+ x
4
+ ... + x
2n-2
divisvel por 1 + x + ... + x
n-1
?
Temos uma diviso de duas somas de PGs finitas, usando a frmula de soma de PG finita e
simplificando elas teremos
| |
| |
1 2k n temos divisvel ser Para
1
1
1
1
.
) 1 ).( 1 (
) 1 ).( 1 (
1
1
) 1 ).( 1 (
) 1 ).( 1 (
teremos, segundo pelo polinmio primeiro o Dividindo
1
1
1
1 ) ( . 1
... 1
) 1 ).( 1 (
) 1 ).( 1 (
1
1 ) ( . 1
... 1
1 2
2
2
2 2 4 2
+ =
+
+
=

+
+
=

+
+


= + + + +
+
+
=


= + + + +

x
x
x
x
x x
x x
x
x
x x
x x
x
x
x
x
x x x
x x
x x
x
x
x x x
n
n
n n
n
n n
n n
n
n n n
n


(b) 1 + x
3
+ x
6
+ ... + x
3n-3
divisvel por 1 + x + ... + x
n-1
?
Temos uma diviso de duas somas de PGs finitas, usando a frmula de soma de PG finita e
simplificando elas teremos
| |
| |
1 3k n temos divisvel ser Para
1
1
1
1
.
) 1 ).( 1 (
) 1 ).( 1 (
1
1
) 1 ).( 1 (
) 1 ).( 1 (
teremos, segundo pelo polinmio primeiro o Dividindo
1
1
1
1 ) ( . 1
... 1
) 1 ).( 1 (
) 1 ).( 1 (
1
1 ) ( . 1
... 1
2
2
2
2 2
2
1 2
2
2
3
3
3 3 6 3
+ =
+ +
+ +
=

+ +
+ +
=

+ +
+ +


= + + + +
+ +
+ +
=


= + + + +

x x
x x
x
x
x x x
x x x
x
x
x x x
x x x
x
x
x
x
x x x
x x x
x x x
x
x
x x x
n
n
n n n
n
n n n
n n
n
n n n n
n

(c) Generalize.
1 entao ... 1 ) ( para ndo, Generaliza
2
+ = + + + + =

pk n x x x x p
p pn p p

QUESTO 16
(a) Resolva a equao 20x
3
- 30x
2
+ 12x - 1 = 0, sabendo-se que1/2 uma de suas razes.
Usando o dispositivo de Briot-Ruffini, temos:
20 -30 12 1
1/2 20 -20 2 0
Ento teremos a equao do 2 grau 2 20 20 ) (
2
+ = x x x p com razes
10
15 5
;
10
15 5
2 1

=
+
= x x

(b) Uma raiz da equao x
3
- (2a+1)x
2
+ a(a+2)x - a(a+1) = 0 a + 1, ache as outras duas.

Usando o dispositivo de Briot-Ruffini, temos:
1 -(2a+1) a(a+2) -a(a+1)
a + 1 1 -a a 0

Ento teremos a equao do 2 grau a ax x x p + =
2
) ( com razes

2
4
;
2
4
2
2
2
1
a a a
x
a a a
x

=
+
=

QUESTO 17
Ache os possveis valores de a Z para que o polinmio a
2
x
4
+ 4x
3
+ 4ax + 7
seja divisvel por x + 1.
Sendo o polinmio divisvel por x+1 ento p(-1)=0, logo:
3
1
0 3 4
0 7 ) 1 ( 4 ) 1 ( 4 ) 1 (
0 ) 1 (
2
1
2
3 4 2
=
=
= +
= + + +
=
a
a
a a
a a
p


QUESTO 18
Prove que todo polinmio de grau 1 irredutvel.
l. irredutve um grau de polinmio todo que concluimos Assim
0. grau de polinmio um teramos outro, do e, um grau de polinmio um temos
lad, um de que, j , impossvel igualdade a tornaria que o , constantes
polinmios seriam e seja, ou , 0 que teramos Portanto,
1 e 1 com ) ( ). (
que tais polinmios existiriam R, em redutvel fosse Se
1. grau de polinmio um Seja
)h(x) f(x) = g(x
h(x) g(x) = gr(h(x)) gr(g(x)) =
= gr(f(x)) gr(h(x)) < = gr(f(x)) gr(g(x)) < x g x f(x) = h
R g(x) f(x), h(x) f(x)
f(x)
e

QUESTO 19
Prove que se f(x) um polinmio de grau maior ou igual a 2 e possui uma raiz real, ento f(x)
redutvel.

redutvel. f(x) Portanto
. 1 de grau o onde , forma da escreve se Ento
. por divisvel logo , 0 ento , de raiz uma Seja
g(x) - a).g(x) f(x) = (x f(x)
x - a f(x) f(a) = f(x) a
>

QUESTO 20
Mostre que todo polinmio f(x) de grau mpar maior ou igual a 3 redutvel.
Se um certo nmero complexo raiz de um polinmio com coeficientes reais, necessariamente seu
conjugado tambm ser. Logo, havendo razes complexas, estas sero em nmero par, existindo,
necessariamente, ao menos uma raiz real "a". Sendo, portanto, o polinmio divisvel por x - a. Ento
ser redutvel.
Obs. Se os coeficientes forem complexos, o Teorema Fundamental da lgebra j garante a existncia
de razes, logo o polinmio ser redutvel.


QUESTO 21
(Critrio de Eisenstein). Seja f(x) = a
0
+ a
1
x +...+a
n
x
n
um polinmio com coeficientes inteiros. Suponha
que exista um primo p tal que:

Ento, f(x) irredutvel sobre Q.
Mostre que os seguintes polinmios f(x) so irredutveis sobre Q.
(a) f(x) = x
4
+ 2x
3
+ 2x
2
+ 2x + 2;
a) 2 no divide 1
b) 2 divide 2, 2, 2 e 2
c) 2
2
no divide 2
f(x) irredutvel sobre Q

(b) f(x) = x
6
+ 15;
a) 3 no divide 1
b) 3 divide 0 e 15
c) 3
2
no divide 15
f(x) irredutvel sobre Q

(c) f(x) = x
4
+ 10x
3
+ 20x
2
+ 30x + 22.
a) 2 no divide 1
b) 2 divide 10, 20, 30 e 22
c) 2
2
no divide 22
f(x) irredutvel sobre Q

QUESTO 22
Determine quais dos polinmios abaixo so irredutveis sobre Q.
Sugesto: Use o critrio de Eisenstein.
(a) x
3
- x + 1
(b) x
3
+ 2x + 10
(c) x
4
- x + 1
O nico polinmio irredutvel sobre Q f(x) = x
3
+ 2x + 10, pois o nmero primo 2 no divide o
coeficiente a
n
, divide os coeficientes a
0
, a
1
, ..., a
n-1
e 2
2
no divide o coeficiente a
0
.

QUESTO 23

QUESTO 24
Determine o polinmio p(x) de grau 7 tal que p(1) = p(2) = ...= p(7) = 8 e p(0) = 1:

8 ) 7 )...( 2 )( 1 (
! 6
1
) ( Portando
! 6
1
! 6 . 7
7
8 ) 7 0 )...( 2 0 )( 1 0 ( 1
: termos ento , 1 ) 0 ( Como
8 7 2 1
: forma da ser deve 7 grau de ) ( polinmio O
+ =
=

= + =
=
x x x x p
A A A
p
) + )...(x- )(x- - p(x) = A(x
x p

Você também pode gostar